Original Changes to Medstudy 2010 Getting Re com

299
Board Review Pediatric Endocrinology Moira C. Pfeifer, MD Pediatric Endocrinologist Pediatric Endocrinologist

Transcript of Original Changes to Medstudy 2010 Getting Re com

Page 1: Original Changes to Medstudy 2010 Getting Re com

Board ReviewPediatric Endocrinology

Moira C. Pfeifer, MD

Pediatric EndocrinologistPediatric Endocrinologist

Page 2: Original Changes to Medstudy 2010 Getting Re com

Overview• Type 2 Diabetes

• Metabolic Syndrome

• Type 1 Diabetes

• Hypoglycemia

• Puberty

• Thyroid

• Calcium/Phosphorus

• Congenital AdrenalHyperplasia

• Ambiguous Genitalia

• Polycystic Ovarian• Puberty

• Growth

• Gynecomastia

• SIADH/DI/Cerebral SaltWasting

• Prolactin

• Polycystic OvarianSyndrome

• Cushing Syndrome

• Adrenal insufficiency

• Pheochromocytoma

• Aldosterone

Page 3: Original Changes to Medstudy 2010 Getting Re com

A parent calls your office concerned thattheir child may have diabetes. Their child isa 2-year-old female with a history of increasedthirst, but no increased urine output. Herweight is stable and her appetite is normal.Her grandmother has T2DM and they checkedthe child’s blood sugar on the grandmother’sthe child’s blood sugar on the grandmother’smeter. The child’s random blood sugarwas 150 mg/dl.

Page 4: Original Changes to Medstudy 2010 Getting Re com

The most appropriate next stepin managing this patient is:

A. Admit to hospital for diabetes education.

B. Give 2 units of insulin aspart (NovoLog®).

C. Reassurance and education concerningthe diagnosis and presentation of diabetes.

D. Have her drink water/check her BG in 2 hours.D. Have her drink water/check her BG in 2 hours.

E. Present to the clinic in the morningfor an oral glucose tolerance test.

Audience Responsenext slide

Page 5: Original Changes to Medstudy 2010 Getting Re com

A parent calls your office concerned that their child mayhave diabetes. Their child is a 2 year old female with ahistory of increased thirst, but no increased urine output.Her weight is stable and her appetite is normal. Hergrandmother has T2DM and they checked her blood sugaron her meter. Her random blood sugar was 150 mg/dl.

The most appropriate next step in managing this pt is:

A. Admit to hospital for diabetes educationA. Admit to hospital for diabetes education

B. Give 2 units of NovoLog

C. Reassurance and education concerning the diagnosisand presentation of diabetes

D. Have her drink water/check her BGin 2 hours

E. Present to the clinic in the morningfor an oral glucose tolerance test

Page 6: Original Changes to Medstudy 2010 Getting Re com

Definition of Diabetes Mellitus• Criteria for diagnosis of diabetes

– Random plasma glucose > 200 mg/dl• +Symptoms of diabetes-polyuria, polydipsia, or

polyphagia– Fasting plasma glucose > 126 mg/dl (fast is defined as no

caloric intake for at least 8 hours)– 2-hour plasma glucose > 200 mg/dl during OGTT (75 g– 2-hour plasma glucose > 200 mg/dl during OGTT (75 g

anhydrous glucose dissolved in water)– HbA1C >6.5% (new criteria as of 2010)

• Insulin– NOT part of the diagnostic criteria– Helpful to get fasting insulin levels for counseling

Diabetes Care January 2010

Page 7: Original Changes to Medstudy 2010 Getting Re com

Diabetes Definitions• Normal Glucose

– Fasting plasma glucose < 100 mg/dL

• Impaired fasting glucose-IFG

– Fasting plasma glucose between 100-125 mg/dL

• Impaired glucose tolerance: IGT

– 2-hr plasma glucose on OGTT > 140 and < 200

• Categories of increased risk for diabetes

– Impaired fasting glucose

– Impaired glucose tolerance

– A1C in the range of 5.7-6.4%

Page 8: Original Changes to Medstudy 2010 Getting Re com

Diabetes Definitions

• Fasting glucose vs. OGTT– ADA-fasting plasma glucose preferred method

• More convenient• Less expensive• Less invasive• Less invasive

–OGTT used in specific situations• Patients with IFG (100-125) should have OGTT• Pt with normal fasting glucose but high suspicion

for T2DM-elevated BMI, AN, positive family historyfor diabetes, non Caucasian, PCOS

Page 9: Original Changes to Medstudy 2010 Getting Re com

Risk Factors for DevelopingType 2 Diabetes Mellitus• Acanthosis Nigricans-AN

– AN is dark, rough skin noted on the neck and axilla• Also seen on fingers, behind knees, inner thighs

– Dark color not due to melanocyte # or content– Dark color due to thickness of keratin superficial

epithelium and hyperplasia of dermis and epidermis– Dark color due to thickness of keratin superficial

epithelium and hyperplasia of dermis and epidermis– 0.5% Caucasians, 6% Hispanic, 13% AA– Rule out insulin resistance– Initial problems

• Increased Insulin, low HDL,increased TG

– Eventual problems• Glucose intolerance, T2DM

Page 10: Original Changes to Medstudy 2010 Getting Re com

Prevention of Type 2 DiabetesMellitus

• ADA screening recommendations for type 2 DM– BMI > 85% for age/sex and 2 other risk factors

• Family history of type 2 in 1st or 2nd degree relative

• Race/ethnicity

– African-American, American Indian

– Hispanic, Asian, Pacific Islander

• Signs of insulin resistance-AN, HTN, dyslipidemia,PCOS, small for gestational age

• Maternal history of DM/GDM during child’s gestation

– Begin at age 10 or onset of puberty due to elevated GHlevels and state of insulin resistance that exists in puberty

– Recommend repeat FBG every three years

Page 11: Original Changes to Medstudy 2010 Getting Re com

Presentation of Type 2 DiabetesMellitus• Type 2 vs. Type 1 DM at presentation

– Antibodies – Lots of overlap

• T1DM: 90% present with at least one antibody –islet, insulin or GAD

• T2DM: up to 30% can be positive for anti-GAD• T2DM: up to 30% can be positive for anti-GADand 35% positive for insulin auto antibodies

• Board answer: get the antibodies todetermine T1DM vs. T2DM

• Clinical answer: time will determine T1DM vs.T2DM

– HLA-DR3, DR4, DQ: strong assoc with type 1

Page 12: Original Changes to Medstudy 2010 Getting Re com

Presentation of Type 2 DiabetesMellitus

• Most asymptomatic

• 33% present with ketonuria w/o DKA

• 5-10% present with DKA

• 94% non-Caucasian• 94% non-Caucasian

• Mean age of diagnosis: 13.8 years

• Familial incidence 45-80% vs. 10% T1DM

• Concordance rates for identical twins nearly 100%for T2DM; only 30-50% for T1DM

• Mean BMI for T2 elevated-up to 85% overweight

• Acanthosis nigricans 60-90% with T2DM vs. 1.2 %T1DM

Page 13: Original Changes to Medstudy 2010 Getting Re com

Therapy for Type 2 Diabetes

• Nutritional therapy/Exercise– Weight maintenance

• Use in children with continued linear growth– Weight reduction

• Age of child—continued linear growth?• Degree of excessive weight• Degree of excessive weight• Association with secondary co morbidities

– HTN, dyslipidemia, apnea, orthopedic• 10% of children can be managed with diet alone

– Work up to 60 minutes of exercise a day

• Medication– Insulin or metformin: only meds FDA approved

in pediatrics

Page 14: Original Changes to Medstudy 2010 Getting Re com

Pharmacologic Therapy forType 2 Diabetes Mellitus• Drug therapy indicated

– Asymptomatic pts who fail to reach glycemic control• Three months after starting lifestyle changes

– Symptomatic patients at presentation• Polyuria, polydipsia, ketosis, A1C > 8.5%• Polyuria, polydipsia, ketosis, A1C > 8.5%

• What type of drugs to use?– 2000: Study by LWPES surveyed 130 members

• 48% started insulin• 44% with oral agents• 71% used metformin• 46% sulfonylurea• 9% thiazolidinediones• 4% meglitinide

Page 15: Original Changes to Medstudy 2010 Getting Re com

Acute Complication of Type 2Diabetes Mellitus

• Nonketotic hyperosmolar state– Syndrome of hyperglycemia & dehydration

– Pathophysiology

• Imbalance between glucose production/excretion

• Acute illness

15

• Acute illness

– Increase glucose production due to stress hormones

– Impairs the capacity to ingest fluids

» Extra cellular fluid and plasma volumes shrink

– Excretion of glucose decreases as u/o decreases

• Ketone production is suppressed-WHY???

– Minimal insulin can suppress ketone production

– Theory is that hyperosmolality inhibits lipolysis

Page 16: Original Changes to Medstudy 2010 Getting Re com

Acute Complications of Type 2Diabetes Mellitus

• Nonketotic hyperosmolar state– Definition

• Glucose > 600 mg/dL

• Serum carbon dioxide > 15 mmol/L

16

• Small ketonuria, absent to low ketonemia

• Serum osmolality > 320 mOsm/kg

• Stupor or coma

– 4% new onset T2DM present with hyperosmolar coma

• Fatality 12%

Page 17: Original Changes to Medstudy 2010 Getting Re com

Acute Complications of Type 2Diabetes Mellitus

• Nonketotic hyperosmolar state– Treatment

• Need to have a high index of suspicion

• Similar risk for cerebral edema as pts with DKA

• Fluid resuscitation over 48 hours

17

• Fluid resuscitation over 48 hours

– Normal saline with potassium to replete half offluid deficit over first 12 hours

– ½ NS with potassium for remainder of fluid

– Add glucose when BG < 250 mg/dl

• Insulin drip

• Antibiotic therapy if indicated

Page 18: Original Changes to Medstudy 2010 Getting Re com

Overview• Type 2 Diabetes

• Metabolic Syndrome

• Type 1 Diabetes

• Hypoglycemia

• Puberty

• Thyroid

• Calcium/Phosphorus

• Congenital AdrenalHyperplasia

• Ambiguous Genitalia

• Polycystic Ovarian• Puberty

• Growth

• Gynecomastia

• SIADH/DI/Cerebral SaltWasting

• Prolactin

• Polycystic OvarianSyndrome

• Cushing Syndrome

• Adrenal insufficiency

• Pheochromocytoma

• Aldosterone

Page 19: Original Changes to Medstudy 2010 Getting Re com

You see a 13-year-old male in the clinic.You notice an increase waistcircumference, a BMI > 95% and a bloodpressure greater than 95% for ageand height. His lipid panel is remarkablefor elevated triglycerides and low HDL.for elevated triglycerides and low HDL.You diagnose metabolic syndrome.

Page 20: Original Changes to Medstudy 2010 Getting Re com

You need to counsel and screenfor which associated diseases?

A. Fatty liver

B. Type 2 diabetes

C. Cardiovascular disease

D. A, B and C

E. Hypothyroidism

Audience Responsenext slide

Page 21: Original Changes to Medstudy 2010 Getting Re com

You see a 13-year-old male in the clinic. You noticean increase waist circumference, a BMI > 95% anda blood pressure greater than 95% for age and height.His lipid panel is remarkable for elevated triglyceridesand low HDL. You diagnosemetabolic syndrome.

You need to counsel and screenYou need to counsel and screenfor which associated diseases?

A. Fatty liverB. Type 2 diabetesC. Cardiovascular diseaseD. A, B and CE. Hypothyroidism

Page 22: Original Changes to Medstudy 2010 Getting Re com

Metabolic syndrome• Adult definition

– Obesity, increased waist circumference, HTN

– Elevated fasting TG and low HDL

– Impaired glucose tolerance, diabetes, or insulinresistance

• Pediatrics: definition not established• Pediatrics: definition not established

– Ford and Li 2008 examined metabolic syndrome in 27publications, found 40 different definitions

• All used blood pressure and lipid profiles

• Most incorporated BMI and glucose measures, butthreshold values varied widely

• Recommended standard definition for metabolicsyndrome in the pediatric population

Page 23: Original Changes to Medstudy 2010 Getting Re com

Metabolic Syndrome• Working definition for pediatric patients

– BP, wt and waist circumference > 85% for age

– Elevated triglycerides and low HDL

• Screening labs

– Lipid panel: TG and HDL– Lipid panel: TG and HDL

• Have high TG and low HDL at presentation

• Acanthosis nigricans and T2DM

– Common association—but not a risk factor norpart of the definition!!

• Elevated fasting insulin and hyperglycemia

• Late finding-not part of initial presentation

Page 24: Original Changes to Medstudy 2010 Getting Re com

Metabolic Syndrome

• Treatment

– Weight loss

– Trials are underway to test the effect of drugs

• Metformin and thiazolidinediones

• Metformin only one FDA-approved in kids

• Adverse conditions associated with metabolic syndrome but• Adverse conditions associated with metabolic syndrome butnot part of the initial definition

– Type 2 diabetes

– Cardiovascular disease

– Fatty liver

– Acanthosis Nigricans

Page 25: Original Changes to Medstudy 2010 Getting Re com

Overview• Type 2 Diabetes

• Metabolic Syndrome

• Type 1 Diabetes

• Hypoglycemia

• Puberty

• Thyroid

• Calcium/Phosphorus

• Congenital AdrenalHyperplasia

• Ambiguous Genitalia

• Polycystic Ovarian• Puberty

• Growth

• Gynecomastia

• SIADH/DI/Cerebral SaltWasting

• Prolactin

• Polycystic OvarianSyndrome

• Cushing Syndrome

• Adrenal insufficiency

• Pheochromocytoma

• Aldosterone

Page 26: Original Changes to Medstudy 2010 Getting Re com

You see a 7-year-old male with T1DM,because his morning blood sugars arealways elevated. He is on 12 NPH anda sliding scale insulin aspart (NovoLog®)in the morning, sliding scale NovoLog®

at dinner and 6 units of NPH at bedtime.at dinner and 6 units of NPH at bedtime.

Page 27: Original Changes to Medstudy 2010 Getting Re com

You recommend:A. Increasing his evening NPH to 8 units

B. Changing his NPH to insulin glargine(Lantus®)

C. Decreasing his evening NPH to 6 units

D. Checking his blood sugar at 0300D. Checking his blood sugar at 0300

E. Changing his sliding scale at dinner

Audience Responsenext slide

Page 28: Original Changes to Medstudy 2010 Getting Re com

You see a 7-year-old male with T1DM because hismorning blood sugars are always elevated. He is on12 NPH and a sliding scale insulin aspart (NovoLog®)in the morning, sliding scale NovoLog at dinner and6 units of NPH at bedtime. You recommend:

A. Increasing his evening NPH to 8 unitsB. Changing his NPH to insulin glargine (Lantus®)B. Changing his NPH to insulin glargine (Lantus®)C. Decreasing his evening NPH to 6 unitsD. Checking his blood sugar at 0300E. Changing his sliding scale

at dinner

Page 29: Original Changes to Medstudy 2010 Getting Re com

Type 1 Diabetes Mellitus• Epidemiology

– Incidence 2/1000 among school-age children

– Increases with age to 1/360 16-year-olds

– Peaks 5-7 YO (infection), puberty (hormones)

– Fall and winter most common time to present

• Natural history– Genetic predisposition + environmental trigger

– Environmental triggers

• Enterovirus, coxsackie B virus, rubella virus,early exposure to milk

– Beta cell destruction

• Clinical onset: 90% of beta cells destroyed

Page 30: Original Changes to Medstudy 2010 Getting Re com

Acute Diabetes Mellitus• DKA: Type 1 or Type 2 Diabetes Mellitus

– 25% initial presentation

– Glucose > 300, pH < 7.3, bicarbonate < 15

– Management

• Normal Saline bolus at 10-20 cc/kg

• Fluid deficit• Fluid deficit

– 1/2 in first 12-24 hours, rest next 24-36 hrs

– 1/2 NS with K until BG < 300—then add D5

• Insulin

– 0.1 u/kg/hr until acidosis improved

• Dextrose

– Add D5W when BG < 250 mg/dl

– Hyperglycemia corrects much faster than acidosis

Page 31: Original Changes to Medstudy 2010 Getting Re com

Acute Diabetes Mellitus• DKA Management

– Bicarbonate administration: Very Controversial

• Never as bolus-risk of cardiac arrhythmia

• Sometimes given with low pH, less than 7

• Complications of DKA• Complications of DKA– Cerebral edema

• Occurs 1-5% of DKA episodes

• Develops 6-12 hrs after initiation of treatment

• Complaints of headache, changedmental status, emesis, delirium, lethargy,incontinence, seizures, pupillary changes,decreasing heart rate, increasing BP

Page 32: Original Changes to Medstudy 2010 Getting Re com

Acute Diabetes Mellitus• Complications of DKA

– Cerebral edema

• Treatment: mannitol and hyperventilation

• Risk factors: < 5 years of age, new-onsetdiabetes, low initial PCO2, high initial serumdiabetes, low initial PCO2, high initial serumurea nitrogen, lesser increasein serum sodium with therapy, treatmentwith bicarbonate

• Can even occur in patient meticulouslytreated for DKA

• Mortality 20%

glucose

sodiumCorrectedsodium

Page 33: Original Changes to Medstudy 2010 Getting Re com

Acute Diabetes Mellitus• Other Complications of DKA

– Hypokalemia: Total body depletion of K+

• Acidosis correction:can result in life-threatening hypokalemia

• Start replacement K+ early

• Hyperkalemia: peaked T waves• Hyperkalemia: peaked T waves

• Hypokalemia: U waves

– Hypoglycemia

– Shock-initial complication!

• Recurrent DKA– Major cause: Noncompliance!

Page 34: Original Changes to Medstudy 2010 Getting Re com

Home management of diabetes

• Meal plan

• Insulin

• Sick day rules

• Follow up labs and consultation• Follow up labs and consultation

Page 35: Original Changes to Medstudy 2010 Getting Re com

Exchange System Food Pyramid

Fat

Grains

Milk Meat

Vegetable Fruit

Page 36: Original Changes to Medstudy 2010 Getting Re com

Meats

Fat

Carbohydrate Counter FoodPyramid

Vegetables

CarbohydratesMilk, fruit, grain

Page 37: Original Changes to Medstudy 2010 Getting Re com

Nutrition Facts

Serving Size ½ Cup

Servings per container 4

Amount per serving

Calories 90 Calories from fat 30

% Daily value

Total Fat 3 g 5%

One carbohydrateExchange = 15 gramsOf carbohydratesTotal Fat 3 g 5%

Saturated Fat 0 g 0%

Cholesterol 0 g 0%

Sodium 300 mg 13%

Total Carbohydrate 13g 4%

Dietary Fiber 3g

Sugars 3g

Of carbohydrates

If you can count it,you can eat it!

Page 38: Original Changes to Medstudy 2010 Getting Re com

Insulin therapy•Types of human insulin

– Rapid• Lispro• Aspart• Glulisine

– Short-acting• Regular insulin• Regular insulin• Semi lente

– Intermediate-acting• NPH• Lente

– Long-acting• Ultralente• Glargine• Levimir

Page 39: Original Changes to Medstudy 2010 Getting Re com

Insulin Therapy• 2 most common insulin regimens

– NPH/ rapid acting—lispro, aspart, glulisine

• Total 0.5-1 u/kg insulin

• 2/3 in a.m., 1/3 in p.m.

• Given as 3 injections: NPH/rapid acting a.m.,rapid acting at dinner, NPH at bedtime

• Diet is restricted and is made to fit the insulin• Diet is restricted and is made to fit the insulinschedule

– No flexibility in how much they can eat orwhen they can eat

• Used in patients that

– Do not want injections at lunch

– Have difficulty with math and countingcarbohydrates

Page 40: Original Changes to Medstudy 2010 Getting Re com

Insulin Therapy• 2 most common insulin regimens

– Lantus/rapid acting insulin

• Using 0.5 to 1 unit/kg—give 60% glargine

• Rest as rapid acting as insulin/carbohydrate ratio

• Diet is not restricted

– Pt can eat as many carbs as they want with eachmeal-they just need to count & cover the carbs

– Pt can eat as many carbs as they want with eachmeal-they just need to count & cover the carbs

– Need to have 2-3 hrs between meals and snacks

– Very flexible diet and schedule

• Used in patients

– Compliant and able to count carbohydrates andfigure out their insulin/carbohydrate ratio

• Insulin pump– Similar to glargine/rapid acting– Only use rapid acting in the pump

Page 41: Original Changes to Medstudy 2010 Getting Re com

Insulin therapy

3 meals-3 spikes of short acting insulin, 4 meals-4 spikes of shortacting insulin, 5 meals-5 spikes of short acting insulin, etc

Page 42: Original Changes to Medstudy 2010 Getting Re com

Diabetes definitions• Honeymoon period

– Residual beta cell function in new-onset IDDM

– Can last months-years; not seen in all patients

– Measure residual function with c-peptide

– Trials to prolong the honeymoon have failed

• Somogyi Phenomenon-“insulin reaction”• Somogyi Phenomenon-“insulin reaction”

– Hypoglycemic reaction classically occurring at night due tocounter regulatory hormone response to low blood sugar(growth hormone, cortisol, epinephrine)

– BG is high in the a.m.-treat by lowering insulin at hs

• Dawn Phenomenon

– Elevated BG in the a.m. due to normal release of GH andwaning effects of insulin-not due to cortisol

– BG is high in a.m. – treat by increasing insulin at hs

– Common in adolescent pts due to GH spikes overnight

Page 43: Original Changes to Medstudy 2010 Getting Re com

Type 1 Diabetes Mellitus• Hemoglobin A1C

– Measurement of 90 days of control

– Infant/toddlers: target HbA1C 7.5-8.5%

– School-age children: target HbA1C < 8%

– Teens: glycemic target HbA1C < 7.5%– Teens: glycemic target HbA1C < 7.5%

– HbA1c of 7% =average BG 170 over 3 months

– Average + 35 mg/dl will result in + 1 point on A1C

• Signs and symptoms of new onset diabetes

– Polyuria, polydipsia, polyphagia, weight loss,emesis…in this order

Page 44: Original Changes to Medstudy 2010 Getting Re com

Complications of diabetes• Neuropathy

– Only Treatment that offers long-term improvementin neuropathy is improvement in diabetes control

– With improvement of control, re growth of smallpain fibers may intensify neuropathic pain for aperiod of time until improvement occursperiod of time until improvement occurs

– ACE inhibitors: no effect on neuropathic pain

– TCA, gabapentin and locally applied capsaicin havehas variable success

Page 45: Original Changes to Medstudy 2010 Getting Re com

Complications of diabetes

• Retinopathy– Present in 20% patients with IDDM

after 10 years and in 45-60% after 20 years

– Screen T1DM after 5 years, T2DM at diagnosis

• Nephropathy– 40% patients with IDDM for 25 years

– Treatment with ACE has slowed down progression

– Causes 50% deaths in adults with IDDM

– Screen for microalbumin after 5 years in T1DM,at diagnosis for T2DM

Page 46: Original Changes to Medstudy 2010 Getting Re com

Diabetes & Celiac Disease• Celiac disease occurs in 1-16% of all pts with T1DM

• More common if they are diagnosed prior to age 10

• ADA recommend screening for celiac soon afterdiagnosis with tissue transglutaminase

• ADA recommends re screening T1DM patients withtissue transglutaminase if they have growth failure,tissue transglutaminase if they have growth failure,failure to gain weight, weight loss or if GI symptomsoccur

• ADA also recommends “Considering re screening ofasymptomatic individuals”

• Patients with positive antibodies: refer to Pediatric GI

Diabetes Care January 2010

Page 47: Original Changes to Medstudy 2010 Getting Re com

Sick day rules for T1DM• General rules

– Patients should check for ketones with BG > 240

– Basic management is fluids and insulin

– Phone vs. clinic management-See immediately with:

• Emesis=acidosis

• Change in mental status• Change in mental status

• Postural symptoms in spite of oral hydration due todehydration

• Moderate or large ketones not clearing withmanagement

• Fever or infectious symptoms

• Unreliable patient or family

• Deep/rapid respirations

Page 48: Original Changes to Medstudy 2010 Getting Re com

Sick days rules for T1DM• Insulin Therapy

– Patient is relatively insulin resistant due to acidemia• Decrease in perfusion contributes to insulin resistance

– Increase in glucose via counter regulatory hormones• Gluconeogenesis and glycogenolysis

– Therefore, even if the patient is not taking adequate po,• They still require insulin…at increased doses!!!!• They still require insulin…at increased doses!!!!

• Fluid therapy-The patient is dehydrated!!!!!– Decrease in intravascular volume/Decrease in

Glomerular filtration rate• Causes an increase serum glucose due to a decrease

in urinary excretion of glucose

Page 49: Original Changes to Medstudy 2010 Getting Re com

Sick days rules for T1DM• Management guidelines

– Educate the family/patient about sick day rules– Follow BG every 3 hours, ketones q void– Talk with patient every 3 hours– Fluids-minimum 8 oz Sugar Free fluid an hour

while awakewhile awake– Food-45 gm carbohydrate every 3 hours (15 gms

an hour)– Insulin

• Insulin sensitivity-rule of 1500– 1500/total insulin=amount one unit will

decrease BG– Establishes that persons sliding scale

• Give novolog/humalog every 3 hours

Page 50: Original Changes to Medstudy 2010 Getting Re com

Overview• Type 2 Diabetes

• Metabolic Syndrome

• Type 1 Diabetes

• Hypoglycemia

• Puberty

• Thyroid

• Calcium/Phosphorus

• Congenital AdrenalHyperplasia

• Ambiguous Genitalia

• Polycystic Ovarian• Puberty

• Growth

• Gynecomastia

• SIADH/DI/Cerebral SaltWasting

• Prolactin

• Polycystic OvarianSyndrome

• Cushing Syndrome

• Adrenal insufficiency

• Pheochromocytoma

• Aldosterone

Page 51: Original Changes to Medstudy 2010 Getting Re com

A 3-year-old female presents to the EDwith seizures due to hypoglycemia. She hadbeen well until yesterday when she startedto have acute gastroenteritis symptoms. Shehas not eaten well since yesterday morning.Her blood sugar at presentation is 35 ng/dL.

Page 52: Original Changes to Medstudy 2010 Getting Re com

Which labs need to be drawn to helpdetermine if her hypoglycemia is due toexogenous administration of insulin?

A. Insulin and c peptide

B. Insulin only

C. C peptide onlyC. C peptide only

D. Insulin and proinsulin

E. Insulin and ketones

Audience Responsenext slide

Page 53: Original Changes to Medstudy 2010 Getting Re com

A 3-year-old female presents to the ED withseizures due to hypoglycemia. She had been welluntil yesterday when she started to have AGEsymptoms. She has not eaten wellsince yesterday morning. Her blood sugarat presentation is 35 ng/dl.

Which labs need to be drawn to help determine if herhypoglycemia is due to exogenous administration of

A. Insulin and c peptide

B. Insulin only

C. C peptide only

D. Insulin and proinsulin

E. Insulin and ketonesAudience Responsenext slide

hypoglycemia is due to exogenous administration ofinsulin?

Page 54: Original Changes to Medstudy 2010 Getting Re com

Hypoglycemia-Pediatrics

Age Whole blood-bedsideglucose

Serum-lab

Prematureinfant

20 mg/dL 25 mg/dLinfant

Full-term infant 40 mg/dL 45 mg/dL

Infant greaterthan 48 hours

40 mg/dL 45 mg/dL

Page 55: Original Changes to Medstudy 2010 Getting Re com

Approach to a child w/ hypoglycemia

• Need to determine the presence or absence ofketones

Hypoglycemia

Acidosis No acidosis

Elevated ketones-Normal-starvationstate-Ketotic hypoglycemia-GSD 3,6,9,10-GH deficiency-Cortisol deficiency--FDPase deficiency

Low ketonesElevated FFA

--Fatty acidoxidationdisorders(normal FFA)

Acidosis No acidosis

Elevated lactate-G6Pase deficiency-Galactosemia

Low ketonesLow FFA-Hyperinsulinism

Langdon DR et al In:Sperling MA, ed. PediatricEndocrinology, 3rd ed. Philadelphia :Saunders Elsevier, 2008

Page 56: Original Changes to Medstudy 2010 Getting Re com

Approach to a child w/hypoglycemia• Need a Critical sample!!!

• Chemistry panel

• Insulin, C-peptide

• Cortisol, growth hormone

• FFA, BOH, acetoacetate

• Lactate, ammonia• Lactate, ammonia

• Total and free carnitine

• Acyl carnitine profile

• Save serum tube

• Urine for ketones and organic acids

• Evaluation of non-hypoglycemic patient

– Infant: 12 hour fast/ Child: 24 hour fast

– Obtain critical sample if hypoglycemic during fast

Sperling, Mark: Pediatric Endocrinology

Page 57: Original Changes to Medstudy 2010 Getting Re com

DDX of a child w/ hypoglycemia

• Hyperinsulinemia

– Most commonly seen in pt on insulin for diabetes

– Munchausen's: factitious, self induced hypoglycemia

– Munchausen's by proxy: causing hypoglycemia inanother person.

– Endogenous insulin excess is really rare…unless it is in– Endogenous insulin excess is really rare…unless it is inthe nursery and pt has congenital hyperinsulinism

• Diagnosis of exogenous hyperinsulinism

• Elevated insulin

• Low C peptide levels

• Diagnosis of endogenous hyperinsulinism

• Elevated insulin

• Elevated C peptide levels

Page 58: Original Changes to Medstudy 2010 Getting Re com

DDX of a child w/ hypoglycemia• Ketotic hypoglycemia

– Classic presentation

• Lean 3-5 year old normal child with an acute illness

• They skip dinner, sleep all night through breakfast

• They have a seizure in the morning after missingbreakfast and dinner

– Etiology of the hypoglycemia– Etiology of the hypoglycemia

• Normal progression during a starvation state

– No glucose for 12-18 hrs; deplete glycogen stores

– Their body moves on to next source of energy-fat-which results in ketone body formation

– Treatment

• Avoid prolonged fasts

• ED evaluation if acutely ill and not taking po

Page 59: Original Changes to Medstudy 2010 Getting Re com

Acute therapy for Hypoglycemia• Conscious patient

– Oral administration is preferred method– Rule of 15

• Eat 15 gms of carbohydrates/wait 15 minutes• Checking Blood Glucose/repeat as needed

– Carbohydrate choices– Carbohydrate choices• Whatever is available!• 15 gms of carbs = one carbohydrate exchange

– One milk, one bread, one fruit exchange• Food labels helpful

Page 60: Original Changes to Medstudy 2010 Getting Re com

Acute therapy for hypoglycemia

• Unconscious patient

– Glucagon

• 1 mg IM

• GI hormone that can induce emesis

• Need appropriate glycogen stores to work• Need appropriate glycogen stores to work

• Parents/patient need to know

– Where it is

– How to administer the med

• Schools not always willing to give glucagon

– IV: 2 ml/kg D10W

• IV therapy is the preferred method of treatmentif the patient already has an IV

Page 61: Original Changes to Medstudy 2010 Getting Re com

Overview• Type 2 Diabetes

• Metabolic Syndrome

• Type 1 Diabetes

• Hypoglycemia

• Puberty

• Thyroid

• Calcium/Phosphorus

• Congenital AdrenalHyperplasia

• Ambiguous Genitalia

• Polycystic Ovarian• Puberty

• Growth

• Gynecomastia

• SIADH/DI/Cerebral SaltWasting

• Prolactin

• Polycystic OvarianSyndrome

• Cushing Syndrome

• Adrenal insufficiency

• Pheochromocytoma

• Aldosterone

Page 62: Original Changes to Medstudy 2010 Getting Re com

An 8-year-old boy presents for a routine physicalexam. On exam, he has Tanner 2 pubic hair and5 cc bilaterally descended testes.

The immediate test that needs to be ordered is:

A. A testicular ultrasoundA. A testicular ultrasoundB. A brain and sella MRIC. A bone ageD. An adrenal CTE. A testosterone level

Audience Responsenext slide

Page 63: Original Changes to Medstudy 2010 Getting Re com

An 8-year-old boy presents for a routine physical exam.On exam, he has Tanner 2 pubic hair and5 cc bilaterally descended testes.

The immediate test that needs to be ordered is:

A. A testicular ultrasoundB. A brain and sella MRIB. A brain and sella MRIC. A bone ageD. An adrenal CTE. A testosterone level

Page 64: Original Changes to Medstudy 2010 Getting Re com

Normal Pubertal Development• Fetus

– GnRH, FSH and LH peak at 20-24 weeks

– Late gestation, levels fall: negative feedbackby the fetoplacental unit

• Infant– Neonate undergoes a mini-puberty– Neonate undergoes a mini-puberty

– FSH and LH rise after birth; girls > boys

– Decrease to prepubertal levels in weeks to months

• Childhood– Dampening of GnRH stimulation

20 weeks

birth

2-6 months

LHFSH

Page 65: Original Changes to Medstudy 2010 Getting Re com

Normal Pubertal Development• Childhood

– Signal that “dampens” GnRH stimulation?

• GABA neurons

• Neuropeptide Y

• Endogenous opioids

• Melatonin• Melatonin

– Interference with inhibitory signal can causeearly puberty

• Congenital CNS disorder

• Acquired CNS disorder = BRAIN TUMOR

Page 66: Original Changes to Medstudy 2010 Getting Re com

Normal Pubertal Development• Adolescent: puberty

– Puberty is a two-part event

• Gonadarche and adrenarche

– Triggers for initiation of puberty: still unknown

• Theories include weight, fat, stress• Theories include weight, fat, stress

• Genetic component for initiation of puberty

– Autosomal Dominant vs polygenic

– Ask the parents when they started puberty

• Higher BMI associated with early puberty in girlsbut not boys

– Gonads

• LH predominance: first indication of puberty

Page 67: Original Changes to Medstudy 2010 Getting Re com

Normal Pubertal Development

Peak GV

menses

Peak GVin females

Peak GVin males

Page 68: Original Changes to Medstudy 2010 Getting Re com

Normal Male Development• First->Gonads-Testes

– Control

• Hypothalamic PituitaryAxis

• Gonadotropins

– Hormone

• Second->Adrenal

– Hormones

• Androstenedione

• DHEAS

• Testosterone

– Control– Hormone

• Testosterone

– Clinical presentation

• First sign of puberty isIncrease testes size

• Eventually have bodyodor, acne, pubic/axillaryhair

– Control

• ?????

• Clinical presentation

– Body odor/Acne

– Pubic/Axillary hair

Page 69: Original Changes to Medstudy 2010 Getting Re com

5 YO male with Pubic Hair

• Testes < 3cc/2.5 cmlength

– Adrenal gland isreleasing adrogens iscausing the PH

• Testes > 3 cc/2.5 cmlength

– Testes are involved inproducing adrogens

– Diagnosis iscausing the PH

– Testes still prepubertaland not involved

– DDX is adrenaldisease-CAH, tumor,Cushings prematureadrenarche, externalexposure toandrogens

– Diagnosis isprecocious pubertyand is a tumor 50% ofthe time

Page 70: Original Changes to Medstudy 2010 Getting Re com

Normal Male Development• First sign of puberty

– Testicular enlargement: Greater than 3 cc/2.5 cm

• Sequence of development is testicular growth,pubarche, penile growth, peak height velocity

• Peak growth velocity Tanner stage 4

• Axillary hair 1-2 years later than pubic hair• Axillary hair 1-2 years later than pubic hair

• LH-Leydig cells: 10% of testicular volume

• FSH-seminiferous tubules: 90% of volume

• Gynecomastia: 65% of males, Tanner 3-4

• Pubertal progression from Tanner 2 to 5 can take2.5 to 5 years to complete

Page 71: Original Changes to Medstudy 2010 Getting Re com

Normal Female Development• First->Gonads-Ovary

– Control

• Hypothalamic PituitaryAxis

• Gonadotropins

– Hormone

• Second->Adrenal

– Hormones

• Androstenedione

• Testosterone

• DHEAS

– Control– Hormone

• Estradiol

– Clinical presentation

• First sign of pubertyis breast development

• Eventually havemenses

– Control

• ???

– Clinical presentation

• Body odor/Acne

• Pubic/Axillary hair

Page 72: Original Changes to Medstudy 2010 Getting Re com

Female pubertal development

• 5 YO girl with pubic hair but no breast development

– Adrenal gland involved

– Breast/gonad not involved

– DDX is adrenal disease-CAH, tumor, cushings,premature adrenarche, external exposure

• 5 YO girl w/breast development but no pubic hair growth• 5 YO girl w/breast development but no pubic hair growth

– Breast/gonad involved

– Adrenal gland not involved

– DDx is excessive estrogen due to external exposure,ovarian tumor, ovarian cyst, precocious puberty

• 5 YO girl with breast and pubic hair development

– DDx is precocious puberty-tumor 10% of the time

Page 73: Original Changes to Medstudy 2010 Getting Re com

Normal Female Development• First sign of puberty: breast enlargement

• Sequence of events is breast development,pubarche, peak height velocity, menarche

• Menarche

– Occurs 2 ½-3 years after breast enlargement

• Peak growth velocity at Tanner 3-4

• Menarche Tanner 4 breast, average age 12.8 years• Menarche Tanner 4 breast, average age 12.8 years

– “Menses” at Tanner 2 breast is NOT menarche

– Physiological leukorrhea commonly precedesmenses by 3-6 months

• With menses: 90% of growth is completed-onaverage grow another 3 inches

• Mean duration of puberty: 4.2 years

Page 74: Original Changes to Medstudy 2010 Getting Re com

Variants of Normal Puberty-Premature Thelarche

• Definition– Isolated breast development

– Females < 6-7 year-old (AA vs. Caucasian)

• Patient profile• Patient profile– Chief complaint: breast development,

lump under breast, chest pain

– BA is normal and within 2SD of CA, BA=HA

– No other signs of puberty, normal GV

– GnRH-FSH predominant-prepubertal

– Normal estradiol levels < 10 pg/ml

Page 75: Original Changes to Medstudy 2010 Getting Re com

Variants of Normal Puberty-Premature Thelarche• Natural History

– Normal pubertal development

– Normal adult height

• Differential diagnosis• Differential diagnosis– Estrogen excess: intrinsic vs. extrinsic

– Rule out early central precocious puberty

– Rule out functional ovarian cysts

– Rule out ovarian tumor

Page 76: Original Changes to Medstudy 2010 Getting Re com

Variants of Normal Puberty-Premature Thelarche

• Evaluation and management– History: estrogen exposure

• Phytoestrogens –”dietary estrogens”-soy, legumes,flax seed, tofu

• Estrogen based creams, hair products, OCP

– Physical exam– Physical exam

• Growth velocity, Tanner stage, skin exam-McCuneAlbright Syndrome

– Lab/X-ray

• Estradiol, LH/FSH, bone age

– Follow-up exam every 3-4 months x 1-2 years

• Monitor for androgen involvement

– Further evaluation if initial screen abnormal

• Head MRI, PUS and GNRH stimulation test

Page 77: Original Changes to Medstudy 2010 Getting Re com

Variants of Normal Puberty-Premature Adrenarche

• Definition

– Body odor, pubic hair, axillary hair, acne

– < 8-year-old females or < 9-year-old males

• Patient profile

– Chief Complaint: PH, AH, BO, acne

– No virilization: no increased GV, no clitoromegaly

– No gonadal involvement

• No breast development or menses in girls

• No testicular enlargement in boys

– Often associated with obesity & CNS insults

Page 78: Original Changes to Medstudy 2010 Getting Re com

Variants of Normal Puberty-Premature Adrenarche• Natural history

– Normal puberty, fertility and growth

– PCOS?• Recent studies suggest girls with premature

adrenarche have increased incidenceadrenarche have increased incidenceof polycystic ovarian syndrome

• Differential diagnosis-rule out excessive androgens– Congenital adrenal hyperplasia

– Tumor-adrenal, gonadal

– Cushing syndrome

– Environmental exposure

Page 79: Original Changes to Medstudy 2010 Getting Re com

Variants of Normal Puberty-Premature Adrenarche• Evaluation and management

– History and physical exam

• Androgen exposure

– Androgen gels used by adult males withhypogonadism can cause hyperandrogenismhypogonadism can cause hyperandrogenismin children if the gel comes in direct contactwith children

– Men using these gels must allow gel to dry onskin, cover with clothes, wash hands withsoap and water prior coming in contact withchildren

• Family history-CAH, PCOS

• Vital signs, Tanner stage, growth velocity

Page 80: Original Changes to Medstudy 2010 Getting Re com

Variants of Normal Puberty-Premature Adrenarche• Evaluation and management

– Lab/x-ray

• DHEAS, androstenedione, testosterone, 17OH

ProgesteroneProgesterone

• Bone age

– Follow-up q 4 months for 1-2 years

– Further evaluation if initial screen abnormal

• ACTH stimulation test –rule out CongenitalAdrenal Hyperplasia

• Adrenal/pelvic imaging-rule out tumor

Page 81: Original Changes to Medstudy 2010 Getting Re com

Variants of Normal Puberty-Premature Testelarche

• No such diagnosis!!!

• Enlarged testes (> 3 cc or 2.5 cm)

– In patient less than 9-year-old is– In patient less than 9-year-old isABNORMAL!!!!!!

• This is an endocrine emergency!!!!

• 50% of patients have a brain tumor!

Page 82: Original Changes to Medstudy 2010 Getting Re com

Precocious Puberty• Previous definition was evidence of any

secondary sexual characteristics before age 8in a female and age 9 in a male

• 1999: Lawson Wilkins Pediatric Endocrine• 1999: Lawson Wilkins Pediatric EndocrineSociety published new recommendations

– Precocious puberty is the appearance of anysecondary sexual characteristics before age 6in an African-American female,7 in Caucasian/nonAfrican American female, 9 in a male

Page 83: Original Changes to Medstudy 2010 Getting Re com

Precocious Puberty• 1999: LWPES recommendations

– Based Pediatric Research in Office Settings(PROS) Network: studied 17000 females

– Not 100% accepted

• Was it breast tissue or adipose tissue?

• Why has menarche stayed the same?• Why has menarche stayed the same?

• Will this miss underlying pathology?

– Different endocrinologists have differentapproaches

• Some use old definition and evaluate if less than8 years old

• Some use new definition and evaluate less than6 (African Americans) or 7 (Caucasian/nonAfrician Americans)

Page 84: Original Changes to Medstudy 2010 Getting Re com

Precocious Puberty• Precocious Puberty

– LWPES recommends evaluation for girls inthe “grey zone”(6-8 year olds) if….

• patient’s pubertal progression is rapid

• patient with neurological problems• patient with neurological problems

• bone age advanced > 2 years

• predicted adult height < 59 in

• family concerned

Page 85: Original Changes to Medstudy 2010 Getting Re com

Precocious Puberty

• Cause

– Brain tumor 10% in females / 50% in males

• Types of tumors

– Hypothalamic hamartoma is most common

• Ectopic neural tissue-GnRH secretory neurons

• Definition of precocious puberty-very specific• Definition of precocious puberty-very specific

– Must have adrenal & gonadal changes in female

– Only need gonadal changes in the male

• Remember: No premature testelarche!!!

• Treatment

– Lupron: nonpulsatile GNRH

– Psychological therapy-may need help coping,often socialize with older children

Page 86: Original Changes to Medstudy 2010 Getting Re com

Syndrome # 1• Café au lait skin pigmentation

• Polyostotic fibrous dysplasia

• Autonomous endocrine hyperfunction

– Missense mutation in the gene codingfor the alpha subunit of the stimulatory G proteinfor the alpha subunit of the stimulatory G protein

– Results in activation in intracellular signalingcascade without hormone stimulation or feedbackloops. Glands work independently

– TSH, FSH, LH, ACTH operatewith the cAMP dependent mechanisms

Page 87: Original Changes to Medstudy 2010 Getting Re com

McCune Albright Syndrome• Café au lait skin pigmentation• Polyostotic fibrous dysplasia• Autonomous endocrine

hyperfunction– Missense mutation in the

gene coding for the alphasubunitsubunitof the stimulatory G protein

– Results in activationin intracellular signalingcascade without hormonestimulation or feedbackloops. Glands workindependently

– TSH, FSH, LH, ACTHoperate with the cAMPdependent mechanisms

Page 88: Original Changes to Medstudy 2010 Getting Re com

Delayed Puberty• Definition

– Absence of secondary sexualcharacteristics in a 13-year-old female ora 14-year-old male

OROR

– If greater than 5 years pass between thebeginning and completion of puberty

Page 89: Original Changes to Medstudy 2010 Getting Re com

Delayed Puberty• Differential diagnosis

– Hypergonadotropic hypogonadism vs. Hypogonadotropichypogonadism

– Random FSH/LH makes the diagnosis

• Hypergonadotropic hypogonadism (high FSH/LH)• Hypergonadotropic hypogonadism (high FSH/LH)

– Gonads are failing

– Pituitary is working overtime

• Hypogonadotropic hypogonadism (low FSH/LH)

– Gonads are fine

– Pituitary is failing

– Need FSH/LH first to establish the etiology and plan yourmanagement

Page 90: Original Changes to Medstudy 2010 Getting Re com

Syndrome # 2

• XXY; 1:1000 males

• Small testes

• Seminiferous tubuledysgenesis

• Behavioral problems

• Below average IQ

• Gynecomastia

• Eunuchoid proportions

Page 91: Original Changes to Medstudy 2010 Getting Re com

Delayed Puberty in the MaleHypergonadotropic Hypogonadism

• Klinefelter Syndrome

– Occurs 1/500 to 1/1000 newborn males

– Classic karyotype is 47,XXY

– Seminiferous tubule dysgenesis-reason testes– Seminiferous tubule dysgenesis-reason testesare small

– Most common male sex chromosomal abnormality

• Presentation of Klinefelter Syndrome

– Prepubertal

• MR, psychiatric problems, fire setting

Page 92: Original Changes to Medstudy 2010 Getting Re com

Delayed Puberty in the MaleHypergonadotropic Hypogonadism

– Pubertal presentation of Klinefelter Syndrome• Tall, slim, testes small for age

• Classic scenario: puberty starts withenlargement of the testes, then stall out withdecrease in testicular sizedecrease in testicular size

• Pubic hair will develop; adrenal gland fine

– Majority develop gynecomastia• 7/10 develop breast cancer

• Must teach breast exam

– Azoospermia and infertility are the norm

– Begin testosterone therapy at age 11-12

Page 93: Original Changes to Medstudy 2010 Getting Re com

Syndrome # 3• Male or female• Web neck• MR 25%• Pulmonary valve

stenosis• Short stature• Pectus excavatum• Increased risk

ALL/CML• HSM• Cubitus valgus• Low factor XI and XII

Page 94: Original Changes to Medstudy 2010 Getting Re com

Delayed Puberty in Male/ FemaleHypergonadotropic Hypogonadism

• Noonan Syndrome– Occurs in boys/girls 1/2000 live births

– Mapped to chromosome 12q

• AD with variable expression

– Similar to Turner Syndrome

• Short stature, web neck

• Pectus carinatum

• Pectus excavatum

• Cubitus valgus

Page 95: Original Changes to Medstudy 2010 Getting Re com

Delayed Puberty in Male/ FemaleHypergonadotropic Hypogonadism

• Noonan Syndrome

– Noonan vs. Turner: Differences

• MR in 25% of Noonan

• Problems in math, problem solving in Turners• Problems in math, problem solving in Turners

• Cardiac defect in Noonan: pulmonic valvestenosis, hypertrophic cardiomyopathy, ASD

• Cardiac defect in Turner: Bicuspid aortic valve:16%; coarctation: 11%; aortic root dissection

– High-frequency sensorineural hearing loss

– Hepatosplenomegaly; Low factor XI and XII

– Increased incidences of ALL and CML

Page 96: Original Changes to Medstudy 2010 Getting Re com

Syndrome # 4

• Retinitispigmentosa

• Obesity

• Polydactyly

• MR• MR

• Hypogonadism

Page 97: Original Changes to Medstudy 2010 Getting Re com

Delayed Puberty in Male/FemaleHypergonadotropic Hypogonadism

• Laurence-Moon-Biedl/Bardet-Biedl Syndrome

– Retinitis pigmentosa, obesity, MR

– Polydactyly, genital hypoplasia, hypogonadism(both hyper and hypogonadotropic hypogonadism)(both hyper and hypogonadotropic hypogonadism)

Page 98: Original Changes to Medstudy 2010 Getting Re com

Syndrome # 5

• Hypogonadotropic hypogonadism

• Anosmia

• Hyposmia

• Migration gene defect due to mutation ofthe KAL gene at Xp22.3the KAL gene at Xp22.3

Page 99: Original Changes to Medstudy 2010 Getting Re com

Delayed Puberty in Male/FemaleHypogonadotropic Hypogonadism

• Kallmann Syndrome– Hyposmia or anosmia

– Gonadotropin deficiency

– Eunuchoid proportions

– X-linked disorder– X-linked disorder

– Mutation of the KAL gene at Xp22.3

• Results in a migration gene defect

Page 100: Original Changes to Medstudy 2010 Getting Re com

Syndrome # 6

• Hypotonia at birth

• Obesity

• Hyperphagia

• MR-IQ 20-80

• Hypogonadism

• Deletion q11-13region ofchromosome 15

Page 101: Original Changes to Medstudy 2010 Getting Re com

Delayed Puberty in Male/FemaleHypogonadotropic Hypogonadism

• Prader Willi Syndrome

– Hypotonia at birth

– Obesity, hyperphagia

– Blond/light brown hair– Blond/light brown hair

– Blue eyes, fair skin

– MR-IQ 20-80

– Small hands and feet

– Small penis, cryptorchidism

– Deletion q11-13 region of chromosome 15

• Paternal-Prader Willi Syndrome

• Maternal-Angelmans Syndrome

Page 102: Original Changes to Medstudy 2010 Getting Re com

Syndrome # 7

• Absence of the septum pellucidum

• Optic disease

• Pituitary disease

Page 103: Original Changes to Medstudy 2010 Getting Re com

Delayed Puberty in Male/FemaleHypogonadotropic Hypogonadism

• Septo optic dysplasia or De Morsier syndrome

– 100% of patient’s presentation

• Agenesis of septum pellucidum

– Eyes-wide spectrum– Eyes-wide spectrum

• Hypoplastic optic nerves, pendular nystagmus

• Visual impairment, blindness

– Endocrine disorders-wide spectrum

• Normal to isolated GH deficiency topanhypopituitarism

Page 104: Original Changes to Medstudy 2010 Getting Re com

Syndrome # 8• 3% of conceptions

– 99% spontaneous Ab

• 45XO most common

• 100% short stature

– absence of one SHOX

• Learningproblemsin math andproblem solving

• Bicuspid aortic– absence of one SHOX(Short HomeobOX)gene

• Short 4th metacarpal

• Classic:swollen hands/ feet atbirth

• Bicuspid aorticvalve

• Congenital renaldisease

• Hypogonadism

Page 105: Original Changes to Medstudy 2010 Getting Re com

Delayed Puberty in the FemaleHypergonadotropic Hypogonadism

• Turner syndrome– 45XO most common: 50%

– 15% mosaic karyotype: 46XX/45X

– Karyotype with a Y chromosome-must removegonads due to risk of malignancygonads due to risk of malignancy

– 1:2000 live births

– 45X present in 3% of conceptions

• 99% spontaneous abortions

– Majority of missing X maternal: Risk does notincrease with increasing maternal age

– Buccal smear is inadequate in identifyingTurner Syndrome or mosaic Turner Syndrome

Page 106: Original Changes to Medstudy 2010 Getting Re com

Delayed Puberty in the FemaleHypergonadotropic Hypogonadism• Turner syndrome

– Short stature

• 100% presentation

• Due to absence of one SHOXgene (Short HomeobOX gene-gene (Short HomeobOX gene-nl one on X and one on the Y)

– Web neck, wide nipples

– Cubitus valgus

– Short 4th metacarpal

– Low posterior hair line

– Nl intelligence: some learningdisabilities in math andproblem solving

Page 107: Original Changes to Medstudy 2010 Getting Re com

Delayed Puberty in the FemaleHypergonadotropic Hypogonadism• Turner syndrome can be diagnosed at 3

different times in a girl’s life

– Birth: swollen hands/feet or with classicphenotype

– Age 3-5 years with decreased growth– Age 3-5 years with decreased growthvelocity/ short stature

– Abnormal puberty

• Majority-abnormal breast developmentbut normal androgen development

• 10-20% with breast development

• Amenorrhea

Page 108: Original Changes to Medstudy 2010 Getting Re com

Delayed Puberty in the FemaleHypergonadotropic Hypogonadism

• Turner syndrome– Cardiac lesions

• Bicuspid aortic valve: 16%; coarctation: 11%

• Risk of aortic root dissection

• Echocardiogram every 5-10 yrs• Echocardiogram every 5-10 yrs

– Renal lesions

• 20% with collection system abnormalities

• 10% with horseshoe kidney

Page 109: Original Changes to Medstudy 2010 Getting Re com

Delay in Pubertal Onset• Constitutional delay-”Late bloomer syndrome”

– Signs and symptoms

• Normal height and weight at birth

• Falls off growth curve at 18-24 months

• Normal growth velocity• Normal growth velocityalong 3-5 %

• Delayed bone age

• Delayed dental age

Page 110: Original Changes to Medstudy 2010 Getting Re com

Delay in Pubertal Onset• Constitutional delay

– Signs and symptoms

• Delayed puberty

– Onset of pubertyaccording to bone age

• Normal final adult height• Normal final adult height

• Positive family history

– Treatment

• Not a candidate for GH therapy

• Low-dose testosterone therapy

• Psychological-may need therapy when dealingwith this diagnosis

Page 111: Original Changes to Medstudy 2010 Getting Re com

Overview• Type 2 Diabetes

• Metabolic Syndrome

• Type 1 Diabetes

• Hypoglycemia

• Puberty

• Thyroid

• Calcium/Phosphorus

• Congenital AdrenalHyperplasia

• Ambiguous Genitalia

• Polycystic Ovarian• Puberty

• Growth

• Gynecomastia

• SIADH/DI/Cerebral SaltWasting

• Prolactin

• Polycystic OvarianSyndrome

• Cushing Syndrome

• Adrenal insufficiency

• Pheochromocytoma

• Aldosterone

Page 112: Original Changes to Medstudy 2010 Getting Re com

You are seeing a 15-year-old male in the clinic.The parents are concerned with his size. He iscurrently at the 5% for height and weight, andhas been growing at this percentile for years.His mid-parental height is 72 inches. He hasan excellent appetite and is very activeafter school. His exam is normal exceptafter school. His exam is normal exceptfor Tanner stage 2 pubic hair and 5 cc testes.His bone age is 13 years.

Page 113: Original Changes to Medstudy 2010 Getting Re com

You recommend:

A. Reassurance and follow-up examin 6 months

B. Testicular U/S

C. Dietary consult

D. FSH/LH

E. Testosterone level

Audience Responsenext slide

Page 114: Original Changes to Medstudy 2010 Getting Re com

You are seeing a 15-year-old male in the clinic.The parents are concerned with his size. He is currentlyat the 5% for height and weight, and has been growingat this percentile for years. His mid-parental height is72 inches. He has an excellent appetite and is veryactive after school. His exam is normal except forTanner stage 2 pubic hair and 5 cc testes. His boneage is 13 years. You recommend:age is 13 years. You recommend:

A. Reassurance and follow-up exam in 6 monthsB. Testicular U/SC. Dietary consultD. FSH/LHE. Testosterone level

Page 115: Original Changes to Medstudy 2010 Getting Re com

Normal Growth• Fetal growth hormone is INSULIN

– Growth hormone is a counter-regulatory hormonein the newborn period

• Responsible for glucose control, not growth!

• 3-year-old children

– Have “picked” their growth percentile

– “Falling off” the growth curve after age 3 isabnormal and requires further evaluation!!!!

– RED FLAG!!!

Page 116: Original Changes to Medstudy 2010 Getting Re com

Normal Growth• Prepubertal growth

– Deceleration of growth prior to puberty

• Pubertal growth– Growth spurt: sex steroids and IGF-1– Growth spurt: sex steroids and IGF-1

– Females

• Growth spurt in Tanner stage 3 of puberty

• Menses: completed 90% of their growth

– Males

• Growth spurt in Tanner stage 4 of puberty

Page 117: Original Changes to Medstudy 2010 Getting Re com

Abnormal growth• Compare height/weight/head circumference

– Decreased height/weight/head circumference

• Premature infants, syndromes

– Normal head circumference, weight > height

• “Short and fat”….ENDOCRINE• “Short and fat”….ENDOCRINE

• Big 3-growth hormone deficiency, hypothyroidism,Cushing syndrome

– Normal head circumference, weight < height

• Short and skinny

• Malnutrition-poor intake/ excessive losses

• Chronic disease-renal, GI

Page 118: Original Changes to Medstudy 2010 Getting Re com

Growth Hormone Deficiency• Idiopathic-most common• Growth-hormone insensitivity

– Increased GH, low IGF1– Can treat with IGF-1

• Septo-optic dysplasia• Cleft lip and palate• Single central incisor• Single central incisor• Craniopharyngioma

– 70% calcified-lateral skull good screen

• Hypothalamic tumor• Ectopic posterior pituitary

– Only seen on MRI– Must evaluate other pituitary hormones if

diagnosed with this disorder

Page 119: Original Changes to Medstudy 2010 Getting Re com

Ectopic Posterior Pituitary

• Problem with development of pituitary

• Anterior pituitary forms from Rathkes pouch-sits in sella

• Posterior pituitary moves down from 3rd ventricle

– Makes infundibulum

– Portal vessels form

– Communication occurs w/hypothalmus

and anterior pituitary

• Ectopic posterior pituitary

– Posterior pituitary never migrates

– No communication with the AP and hypothalamus

– Should have panhypopituitarism

– Some patients are just GH deficient

Page 120: Original Changes to Medstudy 2010 Getting Re com

Syndrome # 9– PTH resistance

• Low Ca+, High Phos

• High PTH

– Phenotype– Phenotype• short stature

• stocky build

• brachymetacarpals

• brachymetatarsals

• round face

• subcutaneouscalcifications

Page 121: Original Changes to Medstudy 2010 Getting Re com

Secondary growth disorders

• Pseudohypoparathyroidism

–Disease with a variety of presentations ofPTH resistance

–Due to imprinting (silencing)of the expressionof GNAS1 gene on chromosome 20 q 13

–Variations of pseudohypoparathyrodism–Variations of pseudohypoparathyrodism

• Pseudohypoparathyroidism Ia, Ib, Ic

• Pseudopseudohypoparathyroidism

• Pseudohypoparathyroidism II

Page 122: Original Changes to Medstudy 2010 Getting Re com

Secondary growth disorders

• Pseudohypoparathyroidism 1a– Parent of origin-mother

– PTH resistance

• Low Ca+, High Phos, High PTH

– Phenotype

• short stature• short stature

• stocky build

• brachymetacarpals

• brachymetatarsals

• round face

• subcutaneous calcifications

Page 123: Original Changes to Medstudy 2010 Getting Re com

Secondary growth disorders• Pseudopseudohypoparathyroidism

– Parent of origin-father

– Pt with pseudohypoparathyroid phenotype

• short stature

• stocky build

• brachymetacarpals

• brachymetatarsals• brachymetatarsals

• round face

• subcutaneous calcifications

– Normal labs

• Normal calcium, phosphorus, PTH

Page 124: Original Changes to Medstudy 2010 Getting Re com

Pseudohypoparathyroidism 1a vspseudopseudohypoparathyroidism

PseudohypoparathyroidismAbnormal labs

PseudopseudohypoparathyroidismNormal labs

PhenotypeAbnormal labs

low calciumhigh phosphorushigh PTH

Normal labsnormal calciumnormal phosphorusnormal PTH

PhenotypeShortstocky

Page 125: Original Changes to Medstudy 2010 Getting Re com

Syndromes associated withshort stature

• Russell-Silver Syndrome– Short stature, frontal bossing, triangular facies,

shortened/incurved 5th metacarpal, asymmetry,Small for Gestational Age

• Congenital growth hormone deficiency– Normal length/weight at birth

– Microphallus: due to IGF-1 deficiency

– Direct hyperbilirubinemia due to sludging fromGrowth Hormone deficiency

– Hypoglycemia due to lack of counterregulatory affects of Growth Hormone

Page 126: Original Changes to Medstudy 2010 Getting Re com

Growth hormone deficiency

• Single centralincisor…suspectcongenital growthhormonedeficiency!!!!!deficiency!!!!!

• Know this for theboards!

Page 127: Original Changes to Medstudy 2010 Getting Re com

Short Stature Evaluation• Evaluation of the growth curve

– Should be relative to the weight and OFC curve

• DDx of short stature: Not just endocrine!!!

• Evaluation of short stature-Do they meet criteriafor Growth Hormone therapy???for Growth Hormone therapy???– History: don’t forget birth history for SGA, congenital

growth hormone deficiency

– CBC/diff, ESR, Chem 10, TFTs, UA, IGF1, IGFBP3,Celiac panel

– X-RAY: Bone age

Page 128: Original Changes to Medstudy 2010 Getting Re com

Short Stature Evaluation• Don’t forget chromosomes for females

• Growth hormone stimulation test

– Patients with nl screen and decreased GV

• Side effects of growth hormone therapy

– Slipped Capital Femoral Epiphysis (SCFE)– Slipped Capital Femoral Epiphysis (SCFE)

– Pseudotumor cerebri

– Transient carbohydrate intolerance

– Transient hypothyroidism

– Scoliosis

– Does not cause cancer

Page 129: Original Changes to Medstudy 2010 Getting Re com

Growth Hormone Therapy• FDA approval for GH therapy-10 indications

– Growth hormone deficient children• GH level <10 ng/dl on GH stimulation test

– Turner Syndrome– Renal osteodystrophy– Prader Willi Syndrome– Prader Willi Syndrome

• 9 deaths associated with GH due to presumedobstructive apnea

– Small for Gestational Age• If they do not catch up after 2 years of age

– Adults with GH deficiency• Prevention of osteoporosis

Page 130: Original Changes to Medstudy 2010 Getting Re com

Growth Hormone Therapy• FDA approval for GH therapy

– Noonan Syndrome– AIDS wasting syndrome– SHOX deficiency

• SHOX is homeodomain protein needed for normalchondrocyte organization

• Turners, Leri Weil Syndrome, 1-2% normal• Turners, Leri Weil Syndrome, 1-2% normal– Non GH deficient pts-Idiopathic short stature

• > 2.25 SD below the mean-1.2% on the growth curve• Unlikely to reach nl ht > 2 SD-3%-4’11” girls, 5’4” boys• “Severe genetic short stature”• On average- obtain another 4-7 cm of growth on GH• Not paid for by insurance companies• GH companies often pick up the cost for 6-12 months

and re appeal the insurance company

Page 131: Original Changes to Medstudy 2010 Getting Re com

Short stature summary

Genetic orFamilial shortstature

Constitutional“Late Bloomer”

Hormone-GH deficiencyHypothyroidismCushings

Growth Velocity Normal Normal Decreased

Family History Short stature Pubertal delay ?Family History Short stature Pubertal delay ?

Bone Age Normal Delayed Delayed

Page 132: Original Changes to Medstudy 2010 Getting Re com

Constitutional Growth Delay

• Normal growth velocity

• Height 3-5%

• Delayed bone age

• Positive family history

• Normal mid parentalheight

• Delayed puberty

Page 133: Original Changes to Medstudy 2010 Getting Re com

Genetic short stature

• Normal growthvelocity

• Height 3-5%

• Normal bone age

• Short mid-parental• Short mid-parentalheight

• Normal puberty

Page 134: Original Changes to Medstudy 2010 Getting Re com

Endocrine Disorders

• Normal-then decreasedgrowth velocity

• Height-variable

• Delayed bone age

• Variable mid-parental• Variable mid-parentalheight

• Puberty-?

Page 135: Original Changes to Medstudy 2010 Getting Re com

Systemic Disease

• First-decreasedweight velocity

• Then-decreased heightvelocity

• Normal to delayed bone• Normal to delayed boneage

• Variable mid-parentalheight

• Puberty-?

Page 136: Original Changes to Medstudy 2010 Getting Re com

Syndromes Associated withTall Stature

• Familial Tall Stature– Most common cause of tall stature in a child

• Soto's Syndrome: cerebral gigantism– Not an endocrine abnormality– Not an endocrine abnormality

– Born > 90th percentile, grows to > 97% for first4-5 years, then slows down to normal rate

Page 137: Original Changes to Medstudy 2010 Getting Re com

Syndromes Associated withTall Stature

• Beckwith-Wiedemann Syndrome– Fetal overgrowth with macroglossia,

Hepatosplenomegaly, nephromegaly, beta cellhyperplasia, hypoglycemia

– Must follow for Wilms and hepatoblastoma• Alpha fetal protein, abdominal and renal US every• Alpha fetal protein, abdominal and renal US every

3 months for 6 years• Overall incidence of developing malignancies in

BWS is 7.5%– Has been mapped to chromosome 11p15

• Klinefelter’s Syndrome

Page 138: Original Changes to Medstudy 2010 Getting Re com

Marfanoid Body HabitusTall stature/Increased arm spanDecreased upper to lower body

ratio/Arachnodactyly

Marfans syndrome

• Autosomal dominant

Homocystinuria

• MR• Autosomal dominant

• Upward lenssubluxation-IQ up

• Aortic dilatation

• MR

• Downward lenssubluxation-IQ down

• Inborn error of aminoacid metabolism

Page 139: Original Changes to Medstudy 2010 Getting Re com

Overview• Type 2 Diabetes

• Metabolic Syndrome

• Type 1 Diabetes

• Hypoglycemia

• Puberty

• Thyroid

• Calcium/Phosphorus

• Congenital AdrenalHyperplasia

• Ambiguous Genitalia

• Polycystic Ovarian• Puberty

• Growth

• Gynecomastia

• SIADH/DI/Cerebral SaltWasting

• Prolactin

• Polycystic OvarianSyndrome

• Cushing Syndrome

• Adrenal insufficiency

• Pheochromocytoma

• Aldosterone

Page 140: Original Changes to Medstudy 2010 Getting Re com

A 16-year-old male presents to your clinic fora physical exam. He has no complaintsat presentation. His history reveals somelearning difficulties at school. On exam,you notice bilateral gynecomastia. His genitalexam is remarkable for Tanner 4 pubic hair and6 cc testes.6 cc testes.

Page 141: Original Changes to Medstudy 2010 Getting Re com

The most appropriate test toobtain to make his diagnosis is:

A. A drug screen for anabolic steroids

B. Prolactin

C. Chromosomes

D. Estradiol

E. Testosterone

Audience Responsenext slide

Page 142: Original Changes to Medstudy 2010 Getting Re com

A 16-year-old male presents to your clinic for a physicalexam. He has no complaints at presentation. Hishistory reveals some learning difficulties at school.On exam, you notice bilateral gynecomastia. Hisgenital exam is remarkable for Tanner 4 pubic hairand 8 cc testes.

The most appropriate test to obtain to makehis diagnosis is:his diagnosis is:

A. A drug screen for anabolic steroidsB. ProlactinC. ChromosomesD. EstradiolE. Testosterone

Page 143: Original Changes to Medstudy 2010 Getting Re com

Gynecomastia• 2/3 of boys will develop gynecomastia

– Pubertal gynecomastia: < 4 cm diameter breast, Tanner2 breast development

• Tanner stage 2, 3 or 4 pubic hair

• Will resolve with time

• No work up needed if diagnosis made correctly witha complete physical exam including tanner stage anda complete physical exam including tanner stage andtesticular exam

– Macrogynecomastia:> 4 cm diameter breast, Tanner 3,4or 5 breast development

• Will not resolve with time

-A good H&P is vital in determining normal vs abnormal andwho needs an evaluation

Page 144: Original Changes to Medstudy 2010 Getting Re com

Gynecomastia

• Pathophysiology of gynecomastia

– Abnormal estrogen/testosterone ratio

– Not a number, rather a concept

• Clinical pearls• Clinical pearls

–Klinefelter: excessive aromatase production/lowtestosterone

–Galactorrhea: think prolactinoma

–Ketoconazole: inhibits testosterone production

Page 145: Original Changes to Medstudy 2010 Getting Re com

Gynecomastia• Increased estrogen

– Liver, kidney, thyroid disease

• Estrogen not metabolized appropriately

– Adrenal tumor, Testicular tumor, Anabolic steroids

• Increased testosterone->increased estrogen

– HCG secreting tumor->looks like LH

• Increased LH->increased testosterone via• Increased LH->increased testosterone vialeydig cells->increased estrogen

– Increased aromatase activity with increasedconversion of testosterone to estrogen

• Obesity

• Low testosterone– Gonadal failure: Klinefelter Syndrome

Page 146: Original Changes to Medstudy 2010 Getting Re com

Gynecomastia• History and Physical Exam

– Chronic disease: liver, thyroid, kidney

– Medications

– Detailed breast exam

• Patients are worried about breast cancer

• Can be asymmetric and be normal• Can be asymmetric and be normal

– Adenopathy

– Abdominal exam: adrenal tumor/mass

– Testicular exam:Tanner stage, masses-rule out testicular tumor

– Secondary sexual development

Page 147: Original Changes to Medstudy 2010 Getting Re com

Gynecomastia• Laboratory evaluation

– Evaluation is indicated for those patients with

• Atypical Tanner stage: stage 1 or 5

• Atypical age: less than 10 or older than 16

• Chronic illness: liver, thyroid, renal• Chronic illness: liver, thyroid, renal

• Abnormal pubertal progression

• Patients requesting surgery– Surgeons will not see them without normal labs

• Macrogynecomastia– Will not resolve with time-these patients need

surgery

Page 148: Original Changes to Medstudy 2010 Getting Re com

Gynecomastia

• Laboratory evaluation– Routine

• Estradiol/ testosterone-ratio

• LH-rule out LH secreting tumor

• DHEAS-screen for adrenal tumor

• HCG-screen for tumor

– Clinical evidence of chronic disease

• TFTS, LFTS, BUN, Cr, urinalysis

– Testes < 3 cm/8 cc & normal adrenal function

• Chromosomes to rule out Klinefelter's

– Galactorrhea

• Prolactin

Page 149: Original Changes to Medstudy 2010 Getting Re com

Gynecomastia• Secondary laboratory evaluation

– Elevated hCG-must rule out tumor

• Testicular U/S

– If normal: Chest film and abdominal CT

– Elevated estradiol-must rule out tumor

• Testicular U/S

– If normal-adrenal CT

– Elevated DHEAS-must rule out tumor

• Adrenal CT

Page 150: Original Changes to Medstudy 2010 Getting Re com

Overview• Type 2 Diabetes

• Metabolic Syndrome

• Type 1 Diabetes

• Hypoglycemia

• Puberty

• Thyroid

• Calcium/Phosphorus

• Congenital AdrenalHyperplasia

• Ambiguous Genitalia

• Polycystic Ovarian• Puberty

• Growth

• Gynecomastia

• SIADH/DI/Cerebral SaltWasting

• Prolactin

• Polycystic OvarianSyndrome

• Cushing Syndrome

• Adrenal insufficiency

• Pheochromocytoma

• Aldosterone

Page 151: Original Changes to Medstudy 2010 Getting Re com

SIADH

• Etiology– Pneumonia, tumors, TB, CF, meningitis

– Head trauma, encephalitis

– Meds: Vincristine and vinblastine

– Primarily seen in ICU

• Clinically act “Sponge like”• Clinically act “Sponge like”– Decreased u/o & increased plasma volume

• Labs– Hyponatremia, low serum osmolality

– Inappropriate high urine osmolality

• Treat– Fluid restriction and treat underlying etiology

Page 152: Original Changes to Medstudy 2010 Getting Re com

Diabetes Insipidus• Central Diabetes insipidus

– Problem with production and or secretion of ADH– Etiology

• Primary/inherited forms-rare– Wolfram syndrome-DIDMOD

» Diabetes insipidus, diabetes mellitus, opticatrophy, deafness

– Septo optic dysplasia– Septo optic dysplasia• Secondary/acquired forms-more common

– Craniopharyngioma– Optic gliomas– Germinomas– Head injury– ID-Sarcoidosis, Histiocytosis, Tuberculosis– Encephalitis

Page 153: Original Changes to Medstudy 2010 Getting Re com

Diabetes Insipidus• Nephrogenic Diabetes Insipidus

– Inability of kidney to recognize/respond to ADH

– Etiology

• Inherited forms: X-linked

• Acquired forms

– Lithium– Lithium

– Polycystic kidney disorder

– Sickle cell anemia

– Chronic pyelonephritis

– Amyloidosis/Sarcoidosis

– Urinary tract obstructions

Page 154: Original Changes to Medstudy 2010 Getting Re com

Diabetes Insipidus• Clinical presentation

– Polyuria and polydipsia: crave ice water– Infants: FTT, poor feeding, high fevers– Older children: enuresis

• Labs– Hypernatremia, hyperosmolar serum– Hypoosmotic dilute urine– Hypoosmotic dilute urine

• Diagnosis– Water-deprivation test: response to ADH?

• Treatment– Central: Desmopressin (DDAVP®)– Nephrogenic: Low-sodium diet, 300 mL/kg/day

water, thiazide or prostaglandin inhibitors

Page 155: Original Changes to Medstudy 2010 Getting Re com

Cerebral salt wasting• Due to hypersecretion of atrial natriuretic peptide

• Seen in pt with CNS disorders

– brain tumors, head trauma, hydrocephalus,neurosurgery, cerebral vascular accidents

• Clinical presentation

– Excessive urine output with Hypovolemia

• Labs

– Hyponatremia

– Elevated urinary sodium excretion (often >150 mEq/L)

– Normal or high uric acid

– Suppressed vasopressin

– Elevated atrial natriuretic peptide concentrations (>20pmol/L

Page 156: Original Changes to Medstudy 2010 Getting Re com

Cerebral Salt Wasting Vs SIADH

• SIADH

– Hyponatremia

– Decreased urineoutput

– Euvolemia

• Cerebral Salt Wasting

– Hyponatremia

– Excessive urine output

– Hypovolemia

– Nl to high uric acid– Euvolemia

– Low uric acid

– Slightly elevated urinesodium concentration

– Elevated vasopressin

– Normal atrialnatriuretic peptideconcentration

– Nl to high uric acid

– Elevated urinarysodium excretion(often >150 mEq/L)

– Suppressedvasopressin

– Elevated atrialnatriuretic peptideconcentrations

Page 157: Original Changes to Medstudy 2010 Getting Re com

Cerebral Salt Wasting VsSIADH

• SIADH

– Treatment is fluidrestriction andtreatment of the causeof the SIADH

• Cerebral salt wasting

– Treatment with IVreplacement of urinevolume with fluid withsodium chloride atof the SIADH

– No need to treat withsalt supplementation

sodium chloride at150–450 mEq/Ldepending on thedegree of salt loss

– Oral saltsupplementation maybe required

Page 158: Original Changes to Medstudy 2010 Getting Re com

Overview• Type 2 Diabetes

• Metabolic Syndrome

• Type 1 Diabetes

• Hypoglycemia

• Puberty

• Thyroid

• Calcium/Phosphorus

• Congenital AdrenalHyperplasia

• Ambiguous Genitalia

• Polycystic Ovarian• Puberty

• Growth

• Gynecomastia

• SIADH/DI/Cerebral SaltWasting

• Prolactin

• Polycystic OvarianSyndrome

• Cushing Syndrome

• Adrenal insufficiency

• Pheochromocytoma

• Aldosterone

Page 159: Original Changes to Medstudy 2010 Getting Re com

Prolactin• Control

– Under tonic hypothalamic inhibition by dopamine

• Prolactin increased with TRH– High prolactin levels can be seen in hypothyroid

pts

• Prolactinoma• Prolactinoma– Most common anterior pituitary tumor

– Symptoms-amenorrhea, headache, galactorrhea in25%, delayed puberty

– Female 2x males

– Best test: pituitary MRI

– Treat: Bromocriptine or surgery

Page 160: Original Changes to Medstudy 2010 Getting Re com

Overview• Type 2 Diabetes

• Metabolic Syndrome

• Type 1 Diabetes

• Hypoglycemia

• Puberty

• Thyroid

• Calcium/Phosphorus

• Congenital AdrenalHyperplasia

• Ambiguous Genitalia

• Polycystic Ovarian• Puberty

• Growth

• Gynecomastia

• SIADH/DI/Cerebral SaltWasting

• Prolactin

• Polycystic OvarianSyndrome

• Cushing Syndrome

• Adrenal insufficiency

• Pheochromocytoma

• Aldosterone

Page 161: Original Changes to Medstudy 2010 Getting Re com

A 14-year-old female presents for a routine physicalexam. She is normal except for a symmetricalgoiter. She has no evidence of hypothyroidism orhyperthyroidism on review of her history oron her physical exam.

An appropriate workup for her would be:

A. Thyroid ultrasoundB. fT4, TSH, thyroid ultrasoundC. fT4, TSH, thyroid antibodiesD. T3, T4, TSH, thyroglobulinE. Thyroid uptake

Audience Responsenext slide

Page 162: Original Changes to Medstudy 2010 Getting Re com

A 14-year-old female presents for a routine physicalexam. She is normal except for a symmetrical goiter.She has no evidence of hypothyroidism orhyperthyroidism on review of her historyor on her physical exam.

An appropriate workup for her would be:

A. Thyroid ultrasoundB. fT4, TSH, thyroid ultrasoundC. fT4, TSH, thyroid antibodiesD. T3, T4, TSH, thyroglobulinE. Thyroid uptake

Page 163: Original Changes to Medstudy 2010 Getting Re com

Thyroid function tests

• TSH-Most sensitive test of thyroid function

– Most common cause of elevated TSH in pt onlevothyroxine…..NONCOMPLIANCE

• Free T4– Measures the free portion of T4-0.0003 of total!– Measures the free portion of T4-0.0003 of total!

– Best test for kids: fT4 and TSH due to highpercentage of secondary hypothyroidism

• Total T4-Measures free T4 and binding globulin

– Falsely elevated T4-Due to elevation of binding globulin

– Falsely low T4-Due to decreased binding globulin

Page 164: Original Changes to Medstudy 2010 Getting Re com

Thyroid Imaging• Thyroid U/S

– Solid vs. cystic nodules

– Single vs. multinodular goiter

• 30-50% of solitary nodules in kidsare malignant

• Thyroid Scan– Use Tc99 to determine hot

vs. cold nodule

• Results in a picture of the thyroid

• 30% of cold nodules are malignant

Page 165: Original Changes to Medstudy 2010 Getting Re com

Thyroid Imaging• Radioiodine Uptake

– Give Iodine 123 and measure % uptake in 24 hrs

– Results in a number, not a picture

– Rarely done-only if need to determine etiology ofhyperthyroidism

– Increased– Increased

• Graves’, hot nodules

– Decreased

• Post-partum thyroiditis

• Subacute thyroiditis

• Thyroiditis factitia

• Amiodarone

Page 166: Original Changes to Medstudy 2010 Getting Re com

Thyroid Imaging• Who needs imaging??????

– Nodular goiter: YES

• U/S, Thyroid scan

– Symmetrical hypothyroid goiter: NO

– Symmetrical euthyroid goiter: NO

– Symmetrical hyperthyroid goiter– Symmetrical hyperthyroid goiter

• Majority of the time….NO

• May need imaging with radioiodine uptakestudy to differentiate Graves’ from othercauses of hyperthyroidism

Page 167: Original Changes to Medstudy 2010 Getting Re com

Symmetrical Goiter

Clinically fT4 TSH ATA Anti

TPO

TSI Imaging

Hypo Yes Yes Yes Yes No No

Hyper Yes Yes Yes Yes Yes No*

Euthyroid Yes Yes Yes Yes No No

Page 168: Original Changes to Medstudy 2010 Getting Re com

Thyroid-Binding Globulin• Increased TBG-falsely increases T4

– OCP, pregnancy

– Meds: tamoxifen, clofibrate, narcotics

– Diseases: hepatitis, biliary cirrhosis

– Free T4 is normal– Free T4 is normal

• Decrease TBG-falsely decreases T4

– Androgens

– Glucocorticoids

– Nephrotic syndrome

– Genetic-TBG deficiency

– Free T4 is normal

Page 169: Original Changes to Medstudy 2010 Getting Re com

Thyroid Binding Globulin• 2 common clinical scenarios and test questions

– Pt with Thyroid Binding Globulin deficiency will failnewborn screen

• Low total T4 but normal free T4

• Normal TSH

– Teenage female with elevated total T4 on lab-why???

• Due to OCP and elevated estrogen

• Results in increased TBG

• Total T4 elevated but free T4 normal, nl TSH

Page 170: Original Changes to Medstudy 2010 Getting Re com

Hashimoto Thyroiditis• Clinical presentation

– Hypothyroid, hyperthyroid or euthyroid

– Positive family history of autoimmune diseases

– Most common cause of goiter in pts > than 6 yr

• Affects 1 in 600 children

– History and physical exam if hypothyroid– History and physical exam if hypothyroid

• Decreased growth velocity, delayed puberty

• Cold intolerance, constipation

• Myxedematous facies

• Dry skin, brittle hair

• Multilobular thyroid-pebbly surface

– Due to lymphocytic infiltration of thyroid

– Also called chronic lymphocytic thyroiditis

Page 171: Original Changes to Medstudy 2010 Getting Re com

Hashimotos Thyroiditis• Labs

– Thyroid antibodies must be presentto diagnose Hashimoto Thyroiditis

• Antithyroperoxidase antibodies

• Antithyroglobulin antibodies

• Antimicrosomal antibodies• Antimicrosomal antibodies

– 4 possible thyroid function tests in Hashimoto’s

• Hypothyroid: low fT4, high TSH

• Compensated hypothyroidism: nl fT4, high TSH

• Hyperthyroid: high fT4, low TSH

• Euthyroid: nl fT4, nl TSH

Page 172: Original Changes to Medstudy 2010 Getting Re com

Hashimoto Thyroiditis

• Management– If gland is smooth/symmetric-no imaging or nuclear

studies needed

– Hypothyroid/ compensated hypothyroid patients

• Treat with levothyroxine

– Euthyroid patients with positive antibodies– Euthyroid patients with positive antibodies

• ? Treatment with levothyroxine-some try to decreasesize of goiter. Not successful

• Follow labs every 6 months

– Hyperthyroid patients

• Treat with propranolol and methimazole

• PTU no longer recommended in kids-risk of liver cancer

Page 173: Original Changes to Medstudy 2010 Getting Re com

Hashimoto Thyroiditis• Management

– To assess TSH response to medication change,must wait 6-8 weeks

– Chronically elevated TSH in a patient onlevothyroxine is non compliance!levothyroxine is non compliance!

– Clinical scenarios: patients on levothyroxine• Elevated TSH, nl fT4-not taking levothyroxine

• Elevated TSH, elevated fT4 – not taking levothyroxineregularly, then takes a lot of medication justprior to the appointment

– Always follow with fT4 and TSH

Page 174: Original Changes to Medstudy 2010 Getting Re com

Other Causes of Hypothyroidism• Subacute thyroiditis

– Viral infection, post URI

– Patient with fever and tender goiter

– Labs

• Elevated ESR

• Cycle of abnormal TFTs• Cycle of abnormal TFTs

–hypothyroid, hyperthyroid, or euthyroid

– Tx with medications appropriate for thyroid dx

• Central hypothyroidism-two presentations– Low fT4 and low TSH-obvious one

– Low fT4 and inappropriately normal TSH

Page 175: Original Changes to Medstudy 2010 Getting Re com

Graves Disease• Positive family history of autoimmune diseases

• Most common cause thyrotoxicosis in pediatrics

• Antibodies– Thyroid Receptor-stimulating immunoglobulin

• Binds to TSH receptor, causing hyperplasia of the• Binds to TSH receptor, causing hyperplasia of thethyroid & overproduction of thyroid hormone

• Called TSI-must be present to diagnose Gravesdisease

– Antithyroglobulin

– Antithyroperoxidase

• Labs: low TSH, elevated thyroid hormones

Page 176: Original Changes to Medstudy 2010 Getting Re com

Graves Disease• Symptoms

– Nervousness: 80%

– Increased appetite: 60%

– Weight loss: 54%

• Some with weight gain

– Heat intolerance: 33%

– Restless sleep: 27%

– Fatigue: 16%

– Headache: 15%• Some with weight gaindue to increasedappetite

– Increased sweating: 49%

– Hyperactivity: 44%

– Palpitations: 34%

– Diarrhea: 13%

– Poor schoolperformance

– Muscle weakness

Page 177: Original Changes to Medstudy 2010 Getting Re com

Graves Disease

• Signs

– Goiter: 99%

– Tachycardia: 83%

– Wide pulse pressure:77%

• Thyroid Bruit: 53%

• Heart murmur: 43%

• Eyelid lag: 22%

77%

– Systolic hypertension:71%

– Exopthalmos: 66%

– Tremor: 61%

Page 178: Original Changes to Medstudy 2010 Getting Re com

Graves Disease therapy options

• #1 Medications– PTU or methimazole

• Both inhibit thyroid hormone production• PTU also blocks conversion of T4 to T3• PTU no longer recommended in kids due to

risk of liver cancer– B-blockers: initially used to control symptoms– First line of therapy in most centers– Requires prolonged therapy of 2-5 years

• Some pts become hypothyroid on methimazole, treated withsynthyroid to “quiet the thyroid down” for a few years

– High likelihood of remission• Shrinkage of the thyroid• Falling antibodies

– Risk of relapse 3-47%

Page 179: Original Changes to Medstudy 2010 Getting Re com

Graves Disease therapy options• #1 Medications

– 5-32% with side effects

• Most common: rash and transientgranulocytopenia < 1500/mm3

• Most serious-Agranulocytosis• Most serious-Agranulocytosis

– Rare but serious side effect: 0.1-0.2%

– Usually occurs within first 3 months

– < 250/mm3

– Fever, sore throat, mouth sores

– Treat with antibiotics, stop medication

Page 180: Original Changes to Medstudy 2010 Getting Re com

Graves Disease therapy options• #2 Radioiodine therapy

– Oral administration of iodine 131

– 50% remission

– Problems

• Majority get post treatment hypothyroidism• Majority get post treatment hypothyroidism

• Risk of leukemia, thyroid cancer,genetic damage?

–50 years of safety to date

Page 181: Original Changes to Medstudy 2010 Getting Re com

Graves Disease therapy options• #3 Subtotal and total thyroidectomy

– Patients who failed medical therapy, relapsed aftercessation of medication, significant drug reaction,large goiter (> 80 gms), young patient, patientsthat do not want radioiodine therapyor severeophthalmopathyophthalmopathy

– Cure rate of 90%

– Complications

• Hypoparathyroidism

• Recurrent laryngeal nerve damage

• Permanent hypothyroidism

Page 182: Original Changes to Medstudy 2010 Getting Re com

Neonatal Graves Disease• Mortality 25% due to high output cardiac failure

• Caused by– Transplacental passage of mom’s TSI

– Persists as long as TSI remains in system

• Incidence• Incidence– Relatively rare

– Reports of 15% of infants of moms with thyrotoxicosishave lab evidence of high thyroid hormoneconcentrations, but only up to 10% of these 15% aresymptomatic

Page 183: Original Changes to Medstudy 2010 Getting Re com

Neonatal Graves Disease• Risk

– Maternal TSI antibodies correlate with risk andseverity of thyrotoxicosis

– Control of mom’s Graves during pregnancy doesnot reduce risk of neonatal thyrotoxicosis unless itis associated with decreases in TSI

• Symptoms at birth• Symptoms at birth– Tachycardia

– Feeding problems/failure to thrive

– Jitteriness, Thyrotoxic stare

– Persistent jaundice

• Follow often prior to 2 week well baby exam

Page 184: Original Changes to Medstudy 2010 Getting Re com

Neonatal Graves Disease• Neonatal thyrotoxicosis reported in infants

born to moms with Graves treated with ablativetherapy years before– Antibodies never go away!

- Ask why is mom on synthyroid-Graves or Hashimotos

• Treatment– Iodine to reduce secretion of thyroid hormone– Iodine to reduce secretion of thyroid hormone

– Methimazole/Propranolol

– Recommend TFTs every 3-4 weeks

• Resolves– 3-12 weeks with clearance of maternal antibodies

– No increased risk of Graves in the babies future if theyhave neonatal graves

Page 185: Original Changes to Medstudy 2010 Getting Re com

Thyroid Cancer• Thyroid nodules in children =malignancy

– Early part of the century-quoted 70% malignant

– Now more like 20% malignant

• Decreased incidence due to being more careful withradiation

– But must ask about radiation exposure

• Risk factors• Risk factors– Solitary nodule

– History of radiation to head or neck

– Rapidly growing nodule that is firm/hard

– Hoarseness or dysphasia

– Family history of MEN-medullary cancer

• Common presentation of thyroid cancer– Inappropriately dx and treated as cervical adenitis

Page 186: Original Changes to Medstudy 2010 Getting Re com

Thyroid Cancer• Nodules = imaging!!!!!

• Work up for thyroid nodules– Ultrasound: solid vs. cystic

– Thyroid scan: hot vs. cold

– Fine needle aspiration vs. surgery

• Clinically controversial…But…Board• Clinically controversial…But…Boardanswer : “most definitive test” and “besttest for definitive diagnosis” is fine needleaspiration

• Clinical management – Due to high percentage ofthyroid cancer, most pediatric patients with thyroidnodule sent to the most experienced thyroidsurgeon for removal

Page 187: Original Changes to Medstudy 2010 Getting Re com

Thyroid-binding globulin deficiency

• 1 in 8000 individuals

• Sex-linked expressed more completely in males

• Low T4/nl TSH on newborn screen

– TBG deficiency (1/8000)

– Hypothalamic pituitary hypothyroidism (1/100000)

– Differentiate the two by getting a fT4– Differentiate the two by getting a fT4

• TBG deficiency

– low T4, nl fT4, nl TSH

• Hypothalamic/pituitary hypothyroidism

– low T4, low fT4, nl or low TSH

• Can also measure thyroid binding globulin

• It is important to identify individuals with TBG deficiencybecause they do not need medication

Page 188: Original Changes to Medstudy 2010 Getting Re com

DDX of Congenital Hypothyroidism• Premature infants

• Full-term infants

– Thyroid dysgenesis 1:4000(most common in U.S.)

• Thyroid dysgenesis

• Thyroid agenesis• Thyroid agenesis

• Ectopic hypothyroid

– Thyroid dyshormonogenesis 1:30,000

– Hypothalamic-pituitary hypothyroidism 1:100,000

– Transient hypothyroidism 1:40,000

• Iodine deficiency

– Most common cause worldwide

Page 189: Original Changes to Medstudy 2010 Getting Re com

Evaluation of Infants WithCongenital Hypothyroidism• Physical exam

– Large posteriorfontanelle > 1 cm

– Prolonged jaundice

• < 5% of patients diagnosedon PE

• 15-20% patients withsuggestive signsat 4-6 weeks of age

– Prolonged jaundice

– Macroglossia

– Hoarse cry

– Distended abdomen

– Hypotonia

– Umbilical hernia

– Goiter

at 4-6 weeks of age

– Progressive disease

Page 190: Original Changes to Medstudy 2010 Getting Re com

Evaluation of Infants WithCongenital Hypothyroidism

• Laboratory

– Confirm diagnosis

•Repeat ft4, T4 and TSH

•Only time I recommend T4-needed along with the fT4 to•Only time I recommend T4-needed along with the fT4 todiagnosis TBG deficiency

– Normal values-Harriet lane

• Diagnostic studies

– Thyroid scan: controversial

•Technetium vs. I-123

– Bone age

Page 191: Original Changes to Medstudy 2010 Getting Re com

Management of Patientswith Congenital Hypothyroidism

• Medication– Start promptly!!...hopefully, w/i first 2 wks of life– Levothyroxine: 10-15 ug/kg/day– Goal is to keep T4 at 75-100% of normal– Goal is to keep T4 at 75-100% of normal– Scored tablets

• There is no suspension available• Can mix in water or breast milk

– Not in soy protein formula• Iron and fiber can also interfere with absorption

Page 192: Original Changes to Medstudy 2010 Getting Re com

Untreated congenitalhypothyroidism

• Irreversible growthfailure and MR

• Feeding problems,constipation

• Hoarse cry

• Dry skin, decreased

• Broad, flat nose

• Pseudo-hypertelorism

• Puffy, myxedematousfacies

• Large, protrudingtongue• Dry skin, decreased

growth of nails and hair

• Delayed tooth eruption

• Protuberant abdomen

• Delayed closureof the anterior/posterior fontanelle

• Cardiomegaly

• Muscular hypertrophy

tongue

• Large fontanelles

• Sparse hair, dry skin

• Hypotonia

• Protuberant abdomenwith umbilical hernia

• Delayed return of thedeep tendon reflexes

Page 193: Original Changes to Medstudy 2010 Getting Re com

Prognosis of PatientsWith Congenital Hypothyroidism• Appropriate evaluation (< 13 days of age)

– Growth and development should normalize

• Some with LD in math and problem solving– Other difficulties: visual spatial tasks, memory, short

attention span, in coordination, hypotonia, hypertonia,attention span, in coordination, hypotonia, hypertonia,speech problems

• Patients treated after 2 weeks– Lose several IQ pts for every week delayed Tx

• 20% with neurosensory hearing deficit

Page 194: Original Changes to Medstudy 2010 Getting Re com

Overview• Type 2 Diabetes

• Metabolic Syndrome

• Type 1 Diabetes

• Hypoglycemia

• Puberty

• Thyroid

• Calcium/Phosphorus

• Congenital AdrenalHyperplasia

• Ambiguous Genitalia

• Polycystic Ovarian• Puberty

• Growth

• Gynecomastia

• SIADH/DI/Cerebral SaltWasting

• Prolactin

• Polycystic OvarianSyndrome

• Cushing Syndrome

• Adrenal insufficiency

• Pheochromocytoma

• Aldosterone

Page 195: Original Changes to Medstudy 2010 Getting Re com

A 12-month-old African American male presents to theclinic for a 12-month well-baby check. He has beenexclusively breastfed. When you plot his growthpercentiles, you notice his height has decreasedfrom the 50% to less than the 5%. The rest of hisgrowth percentiles are stable. You recheck the lengthand confirm that it is correct. Your workup includesa calcium of 7.0 mg/dL (nl 8-10.5 mg/dL),a calcium of 7.0 mg/dL (nl 8-10.5 mg/dL),a phosphorus of 2.0 mg/dL (nl 3.2-6.3 mg/dL),a PTH of 100 pg/ml (nl 10-65 mg/mL),and an alkaline phosphataseof 1000 U/L (nl 150-420 U/L).

Page 196: Original Changes to Medstudy 2010 Getting Re com

His most likely diagnosis is:

A. Hypophosphatemic rickets

B. Vitamin D-deficient rickets

C. Pseudohypoparathyroidism

D. Vitamin D-resistant ricketsD. Vitamin D-resistant rickets

E. Hyperparathyroidism

Audience Responsenext slide

Page 197: Original Changes to Medstudy 2010 Getting Re com

A 12-month-old AA male presents to the clinic for a12 month well baby check. He has been exclusively breastfed. When you plot his growth percentiles, you notice hisheight has decreased from the 50% to less than the 5%.The rest of his growth percentiles are stable. You recheckthe length and confirm that it is correct. Your workupincludes a calcium of 7.0 mg/dL (nl 8-10.5 mg/dL),a phosphorus of 2.0 mg/dL (nl 3.2-6.3 mg/dL), a PTH of100 pg/mL (nl 10-65 mg/mL) and an alkaline phosphatase100 pg/mL (nl 10-65 mg/mL) and an alkaline phosphataseof 1000 U/L (nl 150-420 U/L).

His most likely diagnosis is:A. Hypophosphatemic ricketsB. Vitamin D-deficient ricketsC. PseudohypoparathyroidismD. Vitamin D-resistant ricketsE. Hyperparathyroidism

Page 198: Original Changes to Medstudy 2010 Getting Re com

Calcium/phosphorus metabolism• Both Ca+/Phos high: too much vitamin D

• Both Ca+/Phos low: too little vitamin D

• Ca+ high/ Phos low: hyperparathyroidism

• Ca+ low/Phos high: hypoparathyroidism

Page 199: Original Changes to Medstudy 2010 Getting Re com

Calcium/Phosphorus Metabolism

Phosphorus Calcium

+PTH, +1,25 OHD3

+PTH, +1,25 OHD3

Phosphorus Calcium

- PTH+1,25 OHD3

+PTH,+1,25 OHD3

Page 200: Original Changes to Medstudy 2010 Getting Re com

Vitamin D Metabolismcholesterol

7Dehydro-cholesterol Previtamin D3 Vitamin D3

25 Hydroxylase25 Hydroxylase

25-Hydroxyvitamin D3

1-Hydroxylase

1,25 Dihydroxyvitamin D3

Page 201: Original Changes to Medstudy 2010 Getting Re com

Rickets• Rickets: bone disease of the growth plate

– Under-mineralization of growth plate

– Can only occur in children

• Rickets is result of an abnormality in thenormal Ca/Phos/Vitamin D metabolismnormal Ca/Phos/Vitamin D metabolism– Disrupts normal Calcium/Phosphorus balance

– Leads to secondaryhyperparathyroidism…

• Except for Hypophosphatemic rickets

Page 202: Original Changes to Medstudy 2010 Getting Re com

Clinical presentation of rickets• History

– Irritability, weakness, fractures, growth retardation

• Physical Exam– Infant

• Frontal bossing, craniotabes, widened sutures

• Rachitic rosary, flared wrists

– Older child

• Flared wrists/ankles, genu valgum, genu varum

• Diagnosis– Abnormal labs + abnormal Xrays

Page 203: Original Changes to Medstudy 2010 Getting Re com

Type

Rickets

Ca Phos AlkPhos

25

OHD

1,25OHD

PTH

Vit D Def

Mild Nl, Nl, Nl Nl

Moderate Nl, Nl

Severe

_____Lab Diagnosis_______ ______Etiology_______

Severe

FHR-familialhypophosphatemicrickets

Nl Nl Nl, Nl!!

Deficiency of

25OHase Nl, ?

1 alpha OHase Nl

Resist 1,25 OH Nl

Page 204: Original Changes to Medstudy 2010 Getting Re com

Radiological Changes in Rickets• Irregularity of calcification

• Loss of the provisional zoneof calcification

• Cupping of the metaphysis

• Fraying and widening of the growth plate• Fraying and widening of the growth plate

• Diffuse osteomalacia

Page 205: Original Changes to Medstudy 2010 Getting Re com

Differential Diagnosis of Rickets

• Vitamin D Deficiency-Nutritional Rickets

– Occurs in certain circumstances

– 80-90% Vitamin D made by endogenous production

– Consider this diagnosis in high-risk patient

• Dark-skinned infants• Dark-skinned infants

• Exclusively breastfed for 6 months

• Winter months, inner city, limited sunlight

• Vitamin D-deficient mothers

– Labs

• Low calcium, low phosphorus, high alk phos, highPTH

Page 206: Original Changes to Medstudy 2010 Getting Re com

Therapy for Vitamin D- DeficientRickets

• Vitamin D2-ergocalciferol-calciferol– Liquid: 8000 IU/ml or 200 IU/drop

– Pre hormone: need to take a lot for toxicity

– RDA: 400 IU/day (changing to 800 IU/day??)

– Best option for Vitamin D-deficient rickets– Best option for Vitamin D-deficient rickets

• 2,000 to 3000 IU/day until healed on x-ray

• Takes 3-4 months

• Recommend then changing to RDA

• Transient disease, no long term need for chronicmedications except for a multivitamin

Page 207: Original Changes to Medstudy 2010 Getting Re com

Differential Diagnosis of Rickets• Increased renal loss: hypophosphatemic rickets

– FHR: 1:20,000-most common rickets in NA

– Most commonly X-linked: can also be AD or AR

– Rachitic phenotype expressed more fully in male-be waryof the short, bowed legged grandfather or uncle

– Pathology: phosphorus-wasting– Pathology: phosphorus-wasting

• Decreased renal tubular resorption of phosphorus

– 1,25 Vitamin D-Calcitriol is “normal”

• Inappropriately low for degree of hypophosphatemia

– PTH is normal!!!

• Low phosphorus doesn’t trigger increase in PTH

– Diagnostic labs

• Calcium is normal, serum phosphorus is low, urinephosphorus is high, alkaline phosphatase is slightly high

Page 208: Original Changes to Medstudy 2010 Getting Re com

TX for Hypophosphatemic Rickets• Medications

– Phosphorus supplementation• 4-6 times daily

– Calcitriol therapy• Used to help absorb the phosphorus

• Follow-up– 3-6 months initially, 6-12 months when stable– Alk phos, calcium, phos, x-rays (until healed)– Monitor for side effects of vitamin D

• Hypercalciuria/Nephrocalcinosis• Renal insufficiency/Hyperparathyroidism

Page 209: Original Changes to Medstudy 2010 Getting Re com

Signs and symptoms ofhypocalcemia• Seizures

• Tetany

• Muscular cramping

• Paresthesias-especially around the mouth• Paresthesias-especially around the mouth

• Chvostek sign-abnormal stimulation of the facialnerve resulting in twitching of the facial muscles

• Trousseaus sign-Muscular contraction when using BPcuff-flexion of the wrist, hyperextension of the fingers,and flexion of the thumb

• EKG: prolonged QT interval

Page 210: Original Changes to Medstudy 2010 Getting Re com

Causes of hypocalcemia inneonate

• Early onset: first 72 hours of life-mostlyrelated to birth and pregnancy– Prematurity—most common cause!– Maternal illness

• Diabetes mellitus• Diabetes mellitus• Toxemia• Hyperparathyroidism• Maternal anticonvulsant therapy

– Respiratory distress/Asphyxia– Sepsis/IUGR– Hypomagnesemia

• Usually a transient problem

Page 211: Original Changes to Medstudy 2010 Getting Re com

Causes of hypocalcemia inneonate• Late onset: after 72 hours of life-usually more of a

chronic condition

– High phosphorus load

– Hypoparathyroidism

• Low Ca+, high Phos, low PTH• Low Ca+, high Phos, low PTH

• Transient or Permanent

– Parathyroid hormone resistance

• Low Ca+, high Phos, high PTH

– Vitamin D Deficiency

• Low Ca+, low Phos

– Inherited disorders of Vitamin D Metabolism

• Low Ca+, low Phos

Page 212: Original Changes to Medstudy 2010 Getting Re com

Causes of hypoparathyroidism• DiGeorge Syndrome

– Deletion of chromosome 22q11.2– 1 in 4000 live births– Due to defective development of the third and

fourth pharyngeal pouches– Clinical features

• Facial: mandibular hypoplasia, hypertelorism,short philtrum, malformed or low set ears

• Thymic absence or hypoplasia• Cardiac disease: interrupted aortic arch, ASD,

VSD, pulmonic stenosis, right aortic arch,truncus

• Parathyroid hypoplasia (with hypocalcemia)

Page 213: Original Changes to Medstudy 2010 Getting Re com

Causes of hypoparathyroidism• Hypomagnesemia

– PTH not released and does not work properly in astate of low magnesium

• Autoimmune polyglandular syndrome Type• Autoimmune polyglandular syndrome Type1-also called APS I

– Autosomal recessive

– Candidiasis, hypoparathyroidism, Addison's

– Also see alopecia areata, chronic active hepatitis,vitiligo, diabetes

Page 214: Original Changes to Medstudy 2010 Getting Re com

Signs and symptoms ofhypercalcemia

• Asymptomatic

– Elevated Calcium discovered on routine labs

– Familial hypocalciuric hypercalcemia

• MONES-Skeletal

– Muscular weakness, bone pain– Muscular weakness, bone pain

• STONES-Renal

– Calculus, polydipsia, polyuria, bedwetting

• GRONES-GI

– Anorexia, emesis, constipation, abdominal pain withacute pancreatitis, nausea

• PSYCHOLOGICAL OVERTONES-Neurological

– Drowsiness, confusion, depression, coma, lethargy

Page 215: Original Changes to Medstudy 2010 Getting Re com

Causes of hypercalcemia• Immobilization

– Common board question!!!

– Most common cause in pediatrics

• Hyperparathyroidism

– MEN 1: PPP-Hyperplasia of pancreas,anterior pituitary, parathyroid glandanterior pituitary, parathyroid gland

– MEN 2A: Pheochromocytoma, medullarycarcinoma, hyperparathyroidism

• Familial Hypocalciuric Hypercalcemia

– AD, mutant gene on 3q2

– Elevated Ca+, normal PTH, low urine Ca+

• Granulomatous diseases: sarcoidosis, tuberculosis

• Malignancy: Rare in children

Page 216: Original Changes to Medstudy 2010 Getting Re com

Overview• Type 2 Diabetes

• Metabolic Syndrome

• Type 1 Diabetes

• Hypoglycemia

• Puberty

• Thyroid

• Calcium/Phosphorus

• Congenital AdrenalHyperplasia

• Ambiguous Genitalia

• Polycystic Ovarian• Puberty

• Growth

• Gynecomastia

• SIADH/DI/Cerebral SaltWasting

• Prolactin

• Polycystic OvarianSyndrome

• Cushing Syndrome

• Adrenal insufficiency

• Pheochromocytoma

• Aldosterone

Page 217: Original Changes to Medstudy 2010 Getting Re com

You are called to the nursery to evaluatea baby with ambiguous genitalia. The babyhas clitoromegaly and fused perineum. Thereare no palpable gonads. The baby’s exam isotherwise normal except for hypertension.

Page 218: Original Changes to Medstudy 2010 Getting Re com

The most likely cause of this baby’sambiguous genitalia is:

A. Salt-losing 21 hydroxylase deficiency

B. Nonsalt-losing 21 hydroxylase deficiency

C. 3 beta hydroxysteroiddehydrogenase deficiencydehydrogenase deficiency

D. 11 beta hydroxylase deficiency

E. 23 alpha hydroxylasedeficiency

Audience Responsenext slide

Page 219: Original Changes to Medstudy 2010 Getting Re com

You are called to the nursery to evaluate a baby withambiguous genitalia. The baby has clitoromegaly andfused perineum. There are no palpable gonads. Thebaby’s exam is otherwise normal except forhypertension.

The most likely cause of this baby’s ambiguousgenitalia is:

A. Salt-losing 21 hydroxylase deficiencyB. Nonsalt-losing 21 hydroxylase

deficiencyC. 3 beta hydroxysteroid

dehydrogenase deficiencyD. 11 beta hydroxylase deficiencyE. 23 alpha hydroxylase deficiency

Page 220: Original Changes to Medstudy 2010 Getting Re com

Adrenal Cortex PhysiologyIncreasedACTH

SALT SUGAR PUBERTALHORMONES

Page 221: Original Changes to Medstudy 2010 Getting Re com

21 Hydroxylase Deficiency• Classic 21 OH deficiency

– Etiology 90% of the time– Presents in the newborn period

• 75% salt waster/ 25% non-salt waster• Female: ambiguous genitalia-due to elevated

androgens• Males

– Born with normal genitalia-in spite of elevatedandrogens

– Salt loser» present w/i 2 wks in adrenal crisis

– Non-salt loser» present at 6 months with androgen effects on

physical exam– This is what the state screen is trying to diagnose-

missed males with 21 OH deficiency

Page 222: Original Changes to Medstudy 2010 Getting Re com

Signs/symptoms of 21 OHDeficiency

Salt waster Female Male

Birth

Crisis w/i 2 wks

Ambiguous Normal male

Hypotension

Emesis/diarrhea

Dark skin

Hypoglycemia

Hyponatremia

Hyperkalemia

Increased renin

FTT

Page 223: Original Changes to Medstudy 2010 Getting Re com

Signs/symptoms of 21 OHDeficiency

Non-Saltwaster

Female Male

Birth Ambiguous Normal male

6 months Androgenchanges-PH,AH, BO, acne.Testes areprepubertal <3cc/2.5 cm

Page 224: Original Changes to Medstudy 2010 Getting Re com

Laboratory diagnosis of CAH• 21OH-deficient salt losers

– Hyponatremia, hyperkalemia, hypoglycemia

– Elevated renin, low aldosterone

– Elevated 17OHP

– Elevated androgens– Elevated androgens

– All 3 zones of the adrenal cortex are affected

• 21OH-deficient non-salt losers

– Elevated 17OHP

– Elevated androgens

– Only the glucocorticoids and androgens areaffected-mineralocorticoid zone is intact

Page 225: Original Changes to Medstudy 2010 Getting Re com

Treatment of adrenal crisis• Restore intravascular volume and BP

– 0.9% NaCl without potassium

– Hydrocortisone hemisuccinate (Solu-Cortef®) IV

• Has mineralocorticoid effect so donot need additional mineralocorticoids

– Cortisone acetate-Do not use!– Cortisone acetate-Do not use!

• Poorly soluble, absorbed slowly, won’tresuscitate patient with adrenal insufficiency

• Improve electrolyte abnormalities– Glucose bolus if needed 2 ml/kg D10W

– Glucose in maintenance fluids

– Na/K treated with hydrocortisone bolus

– May need glucose/insulin therapy for hyperkalemia

Page 226: Original Changes to Medstudy 2010 Getting Re com

Early diagnosis of CAH• Newborn screen

– Available since 1977

– 39 states include CAH on newborn screen

– Measures 17OHP in dried blood

– Only screens for 21OH deficiency– Only screens for 21OH deficiency

– Will pick up males with CAH who will bemissed at birth due to normal genitalia

Page 227: Original Changes to Medstudy 2010 Getting Re com

Ambiguous Genitalia due to21OH deficiency

Page 228: Original Changes to Medstudy 2010 Getting Re com

Congenital Adrenal Hyperplasia• 11 B-hydroxylase deficiency

– Virilization & hypertension

– BOARD QUESTION

– Hypertension due to accumulation of DOC

– Plasma DOC and 11-deoxycortisol are high

– Renin and aldosterone suppressed– Renin and aldosterone suppressed

• 3B-HSD deficiency– Abnormal delta 5 steroids/delta 4 steroids

• 17OH pregnenolone/ 17OH progesterone

• DHEA/androstenedione

Page 229: Original Changes to Medstudy 2010 Getting Re com

Adrenal Cortex PhysiologyIncreasedACTH

SALT SUGAR PUBERTALHORMONES

Page 230: Original Changes to Medstudy 2010 Getting Re com

Overview• Type 2 Diabetes

• Metabolic Syndrome

• Type 1 Diabetes

• Hypoglycemia

• Puberty

• Thyroid

• Calcium/Phosphorus

• Congenital AdrenalHyperplasia

• Ambiguous Genitalia

• Polycystic Ovarian• Puberty

• Growth

• Gynecomastia

• SIADH/DI/Cerebral SaltWasting

• Prolactin

• Polycystic OvarianSyndrome

• Cushing Syndrome

• Adrenal insufficiency

• Pheochromocytoma

• Aldosterone

Page 231: Original Changes to Medstudy 2010 Getting Re com

The most common etiologyfor an ambiguous XY baby is:

A. Androgen insensitivityB. Congenital adrenal hyperplasiaC. 5 alpha reductase deficiencyD. Mixed gonadal dysgenesisD. Mixed gonadal dysgenesisE. 17 ketosteroid reductase deficiency

Audience Responsenext slide

Page 232: Original Changes to Medstudy 2010 Getting Re com

The most common etiologyfor an ambiguous XY baby is:

A. Androgen insensitivityB. Congenital adrenal hyperplasiaC. 5 alpha reductase deficiencyD. Mixed gonadal dysgenesisD. Mixed gonadal dysgenesisE. 17 ketosteroid reductase deficiency

Page 233: Original Changes to Medstudy 2010 Getting Re com

Normal Differentiation of the Male

Undifferentiatedgonad

Teste

SRY geneLeydig Cells: releasetestosterone, supportWolffian ducts

gonad

Sertoli Cells: releaseMIS, inhibits formationof Müllerian ducts

Page 234: Original Changes to Medstudy 2010 Getting Re com

Normal Differentiation of the Female

Undifferentiatedgonad

Ovary

No SRY geneNo Testosterone:No support forWolffian ducts

gonad

No Sertoli Cells: NoMIS, Müllerian ductsform

Estrogen not necessary for normal prenatal femalesexual differentiation, but rather the absence of theSRY gene

Page 235: Original Changes to Medstudy 2010 Getting Re com

Female Pseudohermaphroditism• “Overdone female”

• Androgenized external female genitalia

– XX with excessive androgens

• Normal female internal organs

– XX gonad ovary no wolffian ducts due tolack of testosterone no MIS so Müllerian ductssurvive

• “Easier” to diagnose and treat relative to the XYpatient

– DDX is short—too many androgens

– Nearly 100% raised as a female

Page 236: Original Changes to Medstudy 2010 Getting Re com

Female PseudohermaphroditismDifferential diagnosis–excessive androgens

– CAH (mom or baby): most common cause!

– Adrenal tumor

– Danazol—modified androgen used to treatendometriosis-can cause these problems in the XXfetus-Know this! Common board question!fetus-Know this! Common board question!

Know about when the mom is exposed toandrogens and what can occur in the baby

• Exposure 13-18 week gestation-labial fusion

• Exposure after 18th week-clitoral enlargement

• Exposure from 13th week to birth-labial fusion plusclitoral enlargement

Page 237: Original Changes to Medstudy 2010 Getting Re com

Male Pseudohermaphroditism• ”Underdone male:” Genotype XY

– Mild masculinization and palpable gonadal mass

– 4 Defects cause male pseudohermaphroditism

• Defects in testicular differentiation

• Defects in testicular hormones

• Defects in Androgen action

• Unknown causes

– Most common etiology is androgen insensitivity

• 1/20000

• More challenging relative to the XX baby– More difficult to diagnose

– More difficult to treat

– Many raised as a female

Page 238: Original Changes to Medstudy 2010 Getting Re com

Male Pseudohermaphroditism• Defects in Testicular Differentiation

– Denys Drash Syndrome-46 XY

• Pathology-total deficiency of testicular function

• Müllerian ducts are present->no MIS

• Phenotype-Ambiguous genitalia, renal failure atage 3, Wilms tumorage 3, Wilms tumor

– WAGR Syndrome

• Pathology- Missing one copy of 11p13

– Gene carries the Wilms tumor suppressorgene, which is required for testiculardevelopment

• Phenotype-Wilms tumor, aniridia, GUabnormalities, MR

Page 239: Original Changes to Medstudy 2010 Getting Re com

Male Pseudohermaphroditism

• Defects in Testicular Differentiation– SwYeR Syndrome-think SRY!

• Pathology

– XY pure gonadal dysgenesis

– Due to mutation in SRY gene

– No sertoli cells-no MIS– No sertoli cells-no MIS

– No leydig cells-no testosterone

• Gonads are undifferentiated streaks-testosterone levels are low

Page 240: Original Changes to Medstudy 2010 Getting Re com

Male Pseudohermaphroditism

• Defects in Testicular Differentiation– SwYeR Syndrome-think SRY!

• Phenotype-defaults to a female

– Normal stature and complete femalephenotype at birth with vagina, uterus,fallopian tubesfallopian tubes

– Puberty: no breast development ormenstruation due to no estrogen present

• Gonads are at risk for malignancy—25%with gonadoblastoma, so must removesurgically

Page 241: Original Changes to Medstudy 2010 Getting Re com

Swyer Syndrome

Undifferentiatedgonad

Gonadaldysgenesis

XY with mutation inSRY gene Leydig Cells: release

testosterone, supportWolffian ducts

gonad dysgenesis

Sertoli Cells: releaseMIS, inhibits formationof Müllerian ducts

Bottom line-XY female with no breasts or menses and lowTestosterone levels

Page 242: Original Changes to Medstudy 2010 Getting Re com

Male Pseudohermaphroditism• Defects in Testicular Hormones

– Persistent Müllerian Duct Syndrome

• Pathology:

– Absent MIS

– Leydig cell function is normal-normal testosterone

• Phenotype:• Phenotype:

– 46 XY completely virilized male

– Persistent müllerian ducts due to absent MIS

• Classic presentation

– Cryptorchidism

– Surgical correction for cryptorchidism will revealthe uterus and fallopian tubes

• Treatment: removing the müllerian structures

Page 243: Original Changes to Medstudy 2010 Getting Re com

Persistent Mullerian DuctSyndrome

Undifferentiatedgonad

Teste

SRY geneLeydig Cells: releasetestosterone, supportWolffian ducts

gonad

Sertoli Cells: releaseMIS, inhibits formationof Müllerian ducts

Bottom line-XY normal male with mullerian structures found incidentallyon hernia surgery

Board question-describes syndrome, asks what is their chromosomes?

Page 244: Original Changes to Medstudy 2010 Getting Re com

Male Pseudohermaphroditism• Defects in Androgen Action

– Androgen insensitivity syndrome

• Most common cause of malepseudohermaphroditism: 1/20,000

• Pathology

– Defect in androgen receptor gene

• Phenotype-depends on degree of insensitivity

– Complete

» Female external genitalia, blind vaginalpouch, no uterus, intraabdominal testes—primary amenorrhea-because MIS is working

– Incomplete

» Ambiguous genitalia

Page 245: Original Changes to Medstudy 2010 Getting Re com

Male Pseudohermaphroditism• Defects in Androgen Action

– Androgen insensitivity syndrome

• Classic presentation for complete insensitivity

– 1-2% females with inguinal hernia repairdiscover testes in the inguinal canal duringsurgerysurgery

– Primary amenorrhea

• Testosterone levels are elevated dueto presence of the testes

Page 246: Original Changes to Medstudy 2010 Getting Re com

Androgen Insensitivity Syndrome

Undifferentiatedgonad

Teste

SRY geneLeydig Cells: releasetestosterone, supportWolffian ducts

gonad

Sertoli Cells: releaseMIS, inhibits formationof Müllerian ducts

Androgens doNot work due to aReceptor problem

Bottom line-XY with female externalGenitalia (with complete insensitivity),but no female internal organs becauseMIS is working

Page 247: Original Changes to Medstudy 2010 Getting Re com

XY with female phenotype

Swyer syndrome

• Pathology -XY puregondal dysgenesis due toMutation SRY

• Phenotype-female with

Androgen insensitivity

• Pathology-defect inandrogen receptor

• Phenotype-variabledepending on degree of• Phenotype-female with

female external genitalia,uterus, vagina, fallopiantubes

• Gonads undifferentiatedstreaks

• Testosterone levels low

depending on degree ofinsensitivity. Can becomplete female externalphenotype with blindvaginal pouch, but nouterus, ovaries orfallopian tubes

• Gonads are testes

• Testosterone levels high

Page 248: Original Changes to Medstudy 2010 Getting Re com

Rare Chromosome Disorders• 45X/46 XY Gonadal Dysgenesis

– Can present as Turner syndrome

• Must remove gonads due to cancer risk

• Other Turner patients do not need theirstreak gonads removed

– Gonadal tumors – primarily gonadoblastoma –– Gonadal tumors – primarily gonadoblastoma –occur in 25% of patients

• True Hermaphroditism

– Both ovarian and testicular tissue present

– Majority 46XX, 10% XY, rest mosaics

Page 249: Original Changes to Medstudy 2010 Getting Re com

Overview• Type 2 Diabetes

• Metabolic Syndrome

• Type 1 Diabetes

• Hypoglycemia

• Puberty

• Thyroid

• Calcium/Phosphorus

• Congenital AdrenalHyperplasia

• Ambiguous Genitalia

• Polycystic Ovarian• Puberty

• Growth

• Gynecomastia

• SIADH/DI/Cerebral SaltWasting

• Prolactin

• Polycystic OvarianSyndrome

• Cushing Syndrome

• Adrenal insufficiency

• Pheochromocytoma

• Aldosterone

Page 250: Original Changes to Medstudy 2010 Getting Re com

A 16-year-old female presents for evaluationof hirsutism and amenorrhea. She started hermenses at age 12, but has never had regularcycles. She also has acne, and OTCmedication is not treating her acne. On exam,she is obese with severe scarring acne andhirsutism. You suspect polycystic ovarianhirsutism. You suspect polycystic ovariansyndrome.

Page 251: Original Changes to Medstudy 2010 Getting Re com

Initial labs that should bedone include:A. Total and free testosterone, FSH/LH,

DHEAS, 17OHP

B. TFTs, testosterone, prolactin, estradiol

C. FSH/LH, TFTs, cortisol, DHEASC. FSH/LH, TFTs, cortisol, DHEAS

D. 17OHP, estradiol, prolactin, insulin

E. Total and free testosterone,insulin, prolactin, estradiol

Audience Responsenext slide

Page 252: Original Changes to Medstudy 2010 Getting Re com

A 16-year-old female presents for evaluation of hirsutismand amenorrhea. She started her menses at age 12, but hasnever had regular cycles. She also has acne, and OTCmedication is not treating her acne. On exam, she is obesewith severe scarring acne and hirsutism.

You suspect polycystic ovarian syndrome.Initial labs that should be done include:

A. Total and free testosterone, FSH/LH, DHEAS, 17OHPB. TFTs, testosterone, prolactin, estradiolC. FSH/LH, TFTs, cortisol, DHEASD. 17OHP, estradiol, prolactin, insulinE. Total and free testosterone, insulin,

prolactin, estradiol

Page 253: Original Changes to Medstudy 2010 Getting Re com

Polycystic Ovarian Syndrome– Originally described by Stein-Leventhal

– Classic patient

• Bilateral polycystic ovaries

• Amenorrhea

• Hirsutism• Hirsutism

• Insulin resistance

• Obesity

– Clinical evidence of hyperandrogenism-your job is to figure out if it is from theovary (most common) or the adrenal gland

Page 254: Original Changes to Medstudy 2010 Getting Re com

Evaluation ofPolycystic Ovarian Syndrome

• History and physical-s/s of hyperandrogenism– Premature adrenarche– Hirsutism/male pattern alopecia– Acne/acanthosis nigricans– Acne/acanthosis nigricans– Menstrual irregularity– Clitoromegaly– Increased growth velocity

Page 255: Original Changes to Medstudy 2010 Getting Re com

Evaluation ofPolycystic Ovarian Syndrome

• Lab/x-ray to help differentiate ovarian cause vsadrenal cause of hyperandrogenism– Total and Free testosterone

• elevated in PCOS and adrenal disease– DHEAS– DHEAS

• screen for adrenal tumor– LH/FSH

• 3 to 5/1 ratio in PCOS– Androstenedione

• screen for adrenal tumor– 17OH Progesterone

• screen for CAH

Page 256: Original Changes to Medstudy 2010 Getting Re com

Evaluation ofPolycystic Ovarian Syndrome• Lab/x-ray to help differentiate ovarian cause vs

adrenal cause of hyperandrogenism– Prolactin

• screen for prolactinoma as cause of amenorrhea– TFTs

• screen for hypothyroidism as a cause of• screen for hypothyroidism as a cause ofamenorrhea

– Bone age• advanced in hyperandrogenism

– Pelvic U/S• Will need ACTH and/or adrenal imaging if concerned

about adrenal causes of hyperandrogenism– If not concerned about adrenal causes, treated with

OCP and or anti-androgen medication and followedclosely

Page 257: Original Changes to Medstudy 2010 Getting Re com

Overview• Type 2 Diabetes

• Metabolic Syndrome

• Type 1 Diabetes

• Hypoglycemia

• Puberty

• Thyroid

• Calcium/Phosphorus

• Congenital AdrenalHyperplasia

• Ambiguous Genitalia

• Polycystic Ovarian• Puberty

• Growth

• Gynecomastia

• SIADH/DI/Cerebral SaltWasting

• Prolactin

• Polycystic OvarianSyndrome

• Cushing Syndrome

• Adrenal insufficiency

• Pheochromocytoma

• Aldosterone

Page 258: Original Changes to Medstudy 2010 Getting Re com

A 9-year-old female presents to the clinic withconcerns about her weight. Her parents stateshe has gained 20 pounds in the past 8months. She is otherwise a healthy girl, whodenies any chronic illnesses or meds. Herexam is significant for hypertension, centralobesity, striae and muscle weakness. You areobesity, striae and muscle weakness. You areconcerned about Cushing Syndrome.

Page 259: Original Changes to Medstudy 2010 Getting Re com

The best test to screen for CushingSyndrome is:

A. Morning cortisol

B. High-dose dexamethasone suppression test

C. 24-hour urinary free cortisol

D. Low-dose dexamethasone suppression testD. Low-dose dexamethasone suppression test

E. ACTH stimulation test

Audience Responsenext slide

Page 260: Original Changes to Medstudy 2010 Getting Re com

A 9-year-old female presents to the clinic with concernsabout her weight. Her parents state she has gained 20pounds in the past 8 months. She is otherwise a healthygirl who denies any chronic illnesses or meds. Her examis significant for HTN, central obesity, striae and muscleweakness. You are concerned about Cushing Syndrome.

The best test to screen for Cushing Syndrome is:

A. Morning cortisolB. High-dose dexamethasone suppression testC. 24-hour urinary free cortisolD. Low-dose dexamethasone

suppression testE. ACTH stimulation test

Page 261: Original Changes to Medstudy 2010 Getting Re com

Cushing Syndrome• Cushing syndrome

– ACTH-independent Cushing syndrome

– Describes any form of glucocorticoid excess: adrenaltumors, ACTH-independent adrenal hyperplasia, ectopicACTH- producing tumors

– Common cause in patients < 7 years of age

• Cushing disease• Cushing disease

– ACTH-dependent Cushing syndrome

– Refers specifically to pituitary: hypercortisolism due tooverproduction of ACTH

– Common cause in patients > 7 years of age

• Two often lumped together/referred as Cushing Syndrome

– If diagnosed with Cushing Syndrome/cortisol excess,must differentiate ACTH dependent (ie disease)vs.independent (ie syndrome)

Page 262: Original Changes to Medstudy 2010 Getting Re com

Cushing Syndrome

• Signs and symptoms

– Weight gain: 92%

– Growth retardation: 84%

– Osteopenia: 74%

– Fatigue: 67%

– Hirsutism: 46%

– Compulsivebehavior: 44%

– Striae: 36%– Fatigue: 67%

– Hypertension: 63%

– Delayed puberty: 60%

– Buffalo hump: 28%

– Delayed bone age:13%

Page 263: Original Changes to Medstudy 2010 Getting Re com

Cushing syndrome

• Cushing syndrome vs. obesity– Cushing syndrome : poor growth velocity –“short and fat”– Obesity: normal growth velocity – “tall and fat”

• Diagnosis– Delayed bone age– Delayed bone age– Loss of diurnal variation of cortisol

• Normal: AM cortisol high, PM cortisol low• Cushing's: AM cortisol high, PM cortisol high• Insufficiency: AM and PM cortisol low

– 24-hour urinary-free cortisol elevated-best screening test– Obesity can result in elevated cortisol, but is rarely caused

by elevated cortisol…ie cushings• Big problem in adults, not so much in kids

Page 264: Original Changes to Medstudy 2010 Getting Re com

Cushing Syndrome• Further diagnostic testing if unsure why their

cortisol is elevated-cause or affect of their obesity– Low-dose Dexamethasone suppression test

• 2300: give Dexamethasone

• 0800: check cortisol

– < 5 ug/dL: normal-obesity is causing elevated– < 5 ug/dL: normal-obesity is causing elevatedcortisol

– > 5 ug/dL: Cushing syndrome-can’t suppress-cortisol is causing the obesity

– ACTH-dependent or -independent

• CRH stimulation test: CRH will increase ACTH/cortisol in ACTH-dependent Cushing Syndrome

• High-dose Dexamethasone test will suppressACTH-dependent Cushing Syndrome

Page 265: Original Changes to Medstudy 2010 Getting Re com

Cushing Syndrome• Imaging

• CT can detect adrenal tumors > 1.5 cm

• MRI is best test to look for ACTH-secretingadenomas in pituitary

• Syndromes associated with Cushing Syndrome• Syndromes associated with Cushing Syndrome– Carney complex

• Autosomal dominant

• Adrenal gland: pigmented, nodular

• Blue nevi, cardiac, and skin myxomas

• Sexual precocity, thyroid/pituitary tumor

• Melatonin schwannomas

Page 266: Original Changes to Medstudy 2010 Getting Re com

Overview• Type 2 Diabetes

• Metabolic Syndrome

• Type 1 Diabetes

• Hypoglycemia

• Puberty

• Thyroid

• Calcium/Phosphorus

• Congenital AdrenalHyperplasia

• Ambiguous Genitalia

• Polycystic Ovarian• Puberty

• Growth

• Gynecomastia

• SIADH/DI/Cerebral SaltWasting

• Prolactin

• Polycystic OvarianSyndrome

• Cushing Syndrome

• Adrenal insufficiency

• Pheochromocytoma

• Aldosterone

Page 267: Original Changes to Medstudy 2010 Getting Re com

A 14-year-old male presents to the clinicwith fatigue, muscle weakness, and chronicnausea. He has also had some difficultywith walking—he has fallen 3 times in the pastweek. His exam is remarkable for tan skin andabnormal neurological exam, with 4/5 strengthand evidence of ataxia. His labs areand evidence of ataxia. His labs areremarkable for Na 131, K 6.0, and an a.m.cortisol of 3.

Page 268: Original Changes to Medstudy 2010 Getting Re com

The most likely etiology of his adrenalinsufficiency is:

A. Addison disease

B. Adrenoleukodystrophy

C. Congenital adrenal hyperplasia

D. HistoplasmosisD. Histoplasmosis

E. Adrenal hemorrhage

Audience Responsenext slide

Page 269: Original Changes to Medstudy 2010 Getting Re com

A 14-year-old male presents to the clinic with fatigue,muscle weakness, and chronic nausea. He has alsohad some difficulty with walking—he has fallen 3 timesin the past week. His exam is remarkable for tan skinand abnormal neurological exam, with 4/5 strength andevidence of ataxia. His labs are remarkable for Na 131,K 6.0 and an a.m. cortisol of 3. The most likely etiologyof his adrenal insufficiency is:of his adrenal insufficiency is:

A. Addison diseaseB. AdrenoleukodystrophyC. Congenital adrenal hyperplasiaD. HistoplasmosisE. Adrenal hemorrhage

Page 270: Original Changes to Medstudy 2010 Getting Re com

Adrenal insufficiency• Primary vs. secondary insufficiency

– Primary adrenal insufficiency

• Disorders of the adrenal gland

• All sections of the adrenal cortex affected

• Increased pigmentation due to increased POMC (Pro-opiomelanocortin- a precursor polypeptide that codesopiomelanocortin- a precursor polypeptide that codesfor ACTH and MSH) which results in increased ACTHand Melanocyte Stimulating Hormone

• Look for the TAN-buzz word for primary adrenalinsufficiency

– Secondary adrenal insufficiency

• Disorder of hypothalamus or pituitary

• Most common: idiopathic or craniopharyngioma

• Affects ACTH and cortisol

• No increased pigmentation due to low ACTH

Page 271: Original Changes to Medstudy 2010 Getting Re com

Causes of primary adrenalinsufficiency

• Adrenal hemorrhage– Difficult delivery/Asphyxia

• Buzzword….scrotal hematoma

– Waterhouse-Friderichsen syndrome: meningococcemia

– Child abuse– Child abuse

• Medications– Ketoconazole inhibits adrenal enzymes

– Rifampin, phenytoin, phenobarbital: increasemetabolism of corticosteroids

Page 272: Original Changes to Medstudy 2010 Getting Re com

Causes of primary adrenalinsufficiency

• Autoimmune destruction - Addison Disease - 80%

– Type I autoimmune polyendocrinopathy syndrome-APS I

• Autosomal recessive, gene 21q22.3

• First: chronic mucocutaneous candidiasis

• Hypoparathyroidism and Addison next

• Gonadal failure, hair loss, vitiligo, nail disease, chronicactive hepatitis, pernicious anemia, IDDM

• Thyroid disease is rare in type 1

• Male = female, no HLA association

• This is not MEN!

Page 273: Original Changes to Medstudy 2010 Getting Re com

Causes of primaryadrenal insufficiency

• Autoimmune destruction - Addison Disease - 80%

– Type II autoimmune polyendocrinopathy syndrome-APS II

• Autoimmune thyroid disease and T1DM-mostcommonly seen along with Addison disease in Type IIcommonly seen along with Addison disease in Type II

• Females common, 2-3:1 to males

• HLA-D3 and D4

• Can also rarely see vitiligo, alopecia, hypogonadism,pernicious anemia, celiac, RA

• This is not MEN!!!

Page 274: Original Changes to Medstudy 2010 Getting Re com

Causes of primaryadrenal insufficiency

– Infectious: 15%

• TB, HIV, CMV, disseminated fungal

• Coccidiomycosis, histoplasmosis

• Meningococcemia• Meningococcemia

– CAH: salt losing

• 21 OH deficiency, 3Bhydroxysteroiddehydrogenase deficiency, lipoid adrenalhypoplasia

– Infiltrative diseases

• Hemochromatosis, amyloidosis, sarcoidosis

Page 275: Original Changes to Medstudy 2010 Getting Re com

Causes of PrimaryAdrenal Insufficiency

– Congenital adrenal hypoplasia

• Presents in neonatal period/late childhood

• Mainly affects boys

• Mapped to DAX-1 gene on chromosome Xp21• Mapped to DAX-1 gene on chromosome Xp21

• Presents increased pigmentation and crisis

• Cryptorchidism is common

Page 276: Original Changes to Medstudy 2010 Getting Re com

Causes of PrimaryAdrenal Insufficiency• Adrenoleukodystrophy

– Primarily X linked-presents in childhood/adolescent

– Very long-chain fatty acids infiltrate adrenal/cns

– Adrenal Sx can precede or follow neurological Sx– Adrenal Sx can precede or follow neurological Sx

– Classic presentation: Boy 5-15 with behavioralchanges, decreased school performance,abnormal gait, increasing clumsiness

– May also have seizures, ataxia, hypertonia

– Adrenal insufficiency seen in 50% of boys

– Skin hyperpigmentation if adrenal affected

– MRI: Periventricular demyelization

Page 277: Original Changes to Medstudy 2010 Getting Re com

Adrenal insufficiency due towithdrawal of glucocorticoidtherapy

– Nausea, loss of appetite, myalgia,muscle weakness, malaise

– Acute signs include hypotension,decreased pulse pressure, pallor, rapidheart rate

– Mineralocorticoid secretion is preserved,so there is no electrolyte imbalance

Page 278: Original Changes to Medstudy 2010 Getting Re com

Signs and symptoms of PrimaryAdrenal Insufficiency

– Anorexia

– Apathy and confusion

– Dehydration/ postural hypotension

– Hyperkalemia/Hypoglycemia/Hyponatremia

– Hypovolemia and tachycardia

– Nausea and vomiting without diarrhea!

– Salt craving-will often drink pickle juice!

– Weakness

Page 279: Original Changes to Medstudy 2010 Getting Re com

Adrenal Insufficiency

• Laboratory tests– Low cortisol < 5 mcg/dL, especially under stress

– Random cortisol > 20 mcg/dL rules out disease

– ACTH stimulation test definitive diagnosis

• Low cortisol unresponsive to ACTH: diagnostic for• Low cortisol unresponsive to ACTH: diagnostic foradrenal disease

• Low cortisol, which increases with ACTH indicatesendogenous insufficiency of ACTH: diagnostic forcentral disease

– Low Na/elevated K with primary disease only

• Due to loss of aldosterone

Page 280: Original Changes to Medstudy 2010 Getting Re com

Overview• Type 2 Diabetes

• Metabolic Syndrome

• Type 1 Diabetes

• Hypoglycemia

• Puberty

• Thyroid

• Calcium/Phosphorus

• Congenital AdrenalHyperplasia

• Ambiguous Genitalia

• Polycystic Ovarian• Puberty

• Growth

• Gynecomastia

• SIADH/DI/Cerebral SaltWasting

• Prolactin

• Polycystic OvarianSyndrome

• Cushing Syndrome

• Adrenal insufficiency

• Pheochromocytoma

• Aldosterone

Page 281: Original Changes to Medstudy 2010 Getting Re com

What is the primary catecholaminethat is elevated in a childwith a pheochromocytoma?

A. EpinephrineB. NorepinephrineB. NorepinephrineC. CortisolD. AldosteroneE. Testosterone

Audience Responsenext slide

Page 282: Original Changes to Medstudy 2010 Getting Re com

What is the primary catecholaminethat is elevated in a childwith a pheochromocytoma?

A. EpinephrineB. NorepinephrineB. NorepinephrineC. CortisolD. AldosteroneE. Testosterone

Page 283: Original Changes to Medstudy 2010 Getting Re com

Pheochromocytoma• Catecholamine-secreting tumor

– Chromaffin cells

– Majority originate in adrenal medulla

– Can also develop in the sympathetic chain

• Most likely at aorta, at level of inferiormesenteric artery or its bifurcationmesenteric artery or its bifurcation

– Size 1-10 cm, majority on right side of body

• 1 in 5 are bilateral

• Associated disorders– Neurofibromatosis, von Hippel-Lindau, MEN IIA

and IIB, tuberous sclerosis, Sturge-Weber, AtaxiaTelangiectasia

Page 284: Original Changes to Medstudy 2010 Getting Re com

Pheochromocytoma• Signs/symptoms

– Sustained or paroxysmal HTN (adults withparoxysmal HTN), headache, palpitations,dizziness, sweating, emesis, seizures

– Good appetite but will have growth failure

– BP 180-260/120-210

– Eyes: papilledema, exudates, hemorrhages

• Labs– Primary catecholamine norepinephrine

(epinephrine in adults)

– Urinary VMA (epinephrine) and metanephrine(norepinephrine) elevated

Page 285: Original Changes to Medstudy 2010 Getting Re com

Pheochromocytoma• Radiology studies

– Ultrasound, CT or MRI

– Nuclear medicine: MIBG study

• I 131 metaiodobenzylguanidine (MIBG)

• Picked up by chromaffin cells

• Can help localize the disease outside• Can help localize the disease outsideof the adrenal gland

• Surgery– Give alpha- and beta-blockers prior to surgery

to control blood pressure

Page 286: Original Changes to Medstudy 2010 Getting Re com

Overview• Type 2 Diabetes

• Metabolic Syndrome

• Type 1 Diabetes

• Hypoglycemia

• Puberty

• Thyroid

• Calcium/Phosphorus

• Congenital AdrenalHyperplasia

• Ambiguous Genitalia

• Polycystic Ovarian• Puberty

• Growth

• Gynecomastia

• SIADH/DI/Cerebral SaltWasting

• Prolactin

• Polycystic OvarianSyndrome

• Cushing Syndrome

• Adrenal insufficiency

• Pheochromocytoma

• Aldosterone

Page 287: Original Changes to Medstudy 2010 Getting Re com

The syndrome associatedwith hypertension, hypokalemia,low renin and low aldosterone is:

A. Bartter SyndromeB. Liddle SyndromeC. Gitelman SyndromeC. Gitelman SyndromeD. Cushing SyndromeE. Carney Syndrome

Audience Responsenext slide

Page 288: Original Changes to Medstudy 2010 Getting Re com

The syndrome associated with hypertension,hypokalemia, low renin and low aldosterone is:

A. Bartter SyndromeB. Liddle SyndromeC. Gitelman SyndromeD. Cushing SyndromeD. Cushing SyndromeE. Carney Syndrome

Page 289: Original Changes to Medstudy 2010 Getting Re com

Excessive MineralocorticoidSecretion• DDX

– Primary hyperaldosteronism

• All disorders in which there is overproductionof aldosterone independent of therenin-angiotensin system

• Aldosterone secreting adenomas

– Unilateral: primarily seen in girls

– Bilateral: primarily seen in Boys

Page 290: Original Changes to Medstudy 2010 Getting Re com

Excessive MineralocorticoidSecretion• Clinical manifestations

– Some asymptomatic with HTN

– HA, dizziness, enuresis, hypokalemia,nocturia, polydipsia

• Labs– Hypokalemia, low renin, high aldosterone

• Diagnosis-Dexamethasone given first– If aldosterone suppresses—pt with glucocorticoid

suppressible variant of hyperaldosteronism

– If no suppression, do CT to look for adenoma

Page 291: Original Changes to Medstudy 2010 Getting Re com

Excessive MineralocorticoidSecretion• Diagnosis

– If CT negative, adrenal vein catheterization

• High concentration of aldosterone on one side:adenoma

• High concentration of aldosterone on both:• High concentration of aldosterone on both:adrenal hyperplasia

• Treatment– Glucocorticoid suppressible hyperaldosterone:

prednisone

– Adenoma-surgery

– Bilateral adrenal hyperplasia: spironolactone oramiloride

Page 292: Original Changes to Medstudy 2010 Getting Re com

Excessive MineralocorticoidSecretion

• Secondary hyperaldosteronism

– Renin angiotensin system is not suppressed

– Plasma renin levels are high

• Due to nephritic syndrome, CHF, liver cirrhosis• Due to nephritic syndrome, CHF, liver cirrhosis

– Edematous states with hyperaldosteronism

• Due to renal artery stenosis, Wilms tumor,juxtaglomerular cell tumors

– Produce excessive renin and thusincreasing aldosterone release

Page 293: Original Changes to Medstudy 2010 Getting Re com

Mineralocorticoid Syndromes• Bartter Syndrome

– Hyperaldosteronism, increased renin

– No hypertension

– Seen in neonate with severe dehydration afterpolyhydramnios

– Increased calcium in urine, stones

• Gitelman Syndrome

– Hyperaldosterone, high renin, normal BP

– Older child with hypocalcemia, hypomagnesium

• Liddle Syndrome-Autosomal dominant

– HTN, hypokalemia, low renin and low aldosterone

– Tx-odium restriction and potassium-sparing diuretic

Page 294: Original Changes to Medstudy 2010 Getting Re com

Bibliography1. Diabetes Care 31: S12-S54, 2010

2. Diabetes Care 31 S31-54, 20103. Nathan, DM et al: Impaired fasting glucose and impaired

glucose tolerance: implications for care.Diabetes Care 30:753-759, 2007

4. Diabetes Care 31:S12-S54, 20085. Kaufman, F: Type 2 Diabetes in Children and Youth.

Endocrinology Clinics of North America 34, September 2005Endocrinology Clinics of North America 34, September 20056. Type 2 Diabetes in children and adolescents: consensus

statement. Diabetes Care 2000: 23, pp 345-3547. Silverstein J: Treatment of type 2 diabetes mellitus

in children and adolescents.J Pediatric Endocrinology Metabol 13:1-7, 2000

8. The National Cholesterol Education Program (NCEP) ExpertPanel on Detection, Evaluation and Treatment of High BloodCholesterol in Adults. JAMA 2001:285, 2486-97

Page 295: Original Changes to Medstudy 2010 Getting Re com

9. Ford, ES, Li, C: Defining the Metabolic Syndromein Children and Adolescents: Will the real DefinitionPlease Stand Up? J Pediatrics 2008; 152:160-4

10. Pediatric Endocrinology, 2nd Edition, Mark A Sperling,MD. Saunders 2002

11. Glaser, N: Risk factors for the Development of CerebralEdema in Children with diabetic Ketoacidosis.NEJM 344:264, 2001

Bibliography

NEJM 344:264, 200112. Vajo, Z: Genetically engineered insulin analogs:

Diabetes in the New Millennium.Pharmacol Rev 52:1, 2000

13. The First Step in Diabetes MealPlanning, The American DiabetesAssociation and The AmericanDietetic Association, 1997

Page 296: Original Changes to Medstudy 2010 Getting Re com

14. Carbohydrate Counting-Level 2 and 3. American DiabetesAssociation, American Dietetic Association. 1997

15. The Diabetes Control and Complications Trial ResearchGroup. Diabetes Care 23. 2000

16. Nathan, B, Palmert, M: Regulation and Disorders of PubertalTiming. Endocrinology and Metabolism Clinics 34: 2005

17. Muir, A. Precocious Puberty. Pediatrics in Review2006;27:373-380

Bibliography

2006;27:373-38018. Diamantopoulos, S. Gynecomastia and Premature Thelarche.

Pediatrics in Review. 2007; 9:e57-e68.19. Rosen, D. Delayed Puberty. Pediatrics in Review.

2001;22:309-31420. Jones, K. Smith’s Recognizable Patterns

of Human Malformation. 4th Edition.Philadelphia. W. B. Saunders Co., 1988

Page 297: Original Changes to Medstudy 2010 Getting Re com

21. Bondy, C: Care of Girls and Women with Turner Syndrome:A Guideline of the Turner Syndrome Study Group. JCEM 92:(1). 2007

22. Rose, S. A General Pediatric Approach to Evaluating a ShortChild. Pediatrics in Review. 2005; 26:410-419

23. Carlson, H, Narula, H: Gynecomastia. Endocrinology andMetabolism Clinics:36, (2). 2007

24. Farber, R, Kerrigan, J: The Multiple Indications for GrowthHormone Treatment of Pediatric Patients. Pediatric Annuals

Bibliography

Hormone Treatment of Pediatric Patients. Pediatric Annuals35:12. 2006

25. Muir. A. Precocious Puberty. Pediatrics in Review. 2006; 27:373-380

26. Rose, S: A General Pediatric Approachto Evaluating a Short Child.Pediatrics in Review. 2005; 26:410-420.

Page 298: Original Changes to Medstudy 2010 Getting Re com

27. Update of guidelines for the use of growth hormonein children: the Lawson Wilkins pediatric endocrinology societydrug and therapeutics committee. JPeds 143:4, 2003

28. Devoe DJ, Miller WL, Conte, FA, et al: Long-term outcome ofchildren and adolescents Following transsphenoidal surgeryfor Cushing disease. J Clin Endocrinol Metab 82:3196, 1997

29. Foley, T. Hypothyroidism. Pediatrics in Review. 2004;25:94-100

Bibliography

25:94-100

30. Halac I, Zimmerman D: Thyroid Nodules and Cancersin Children. Endocrinology and Metabolism Clinics 34:3. 2005

31. Hanna, C, LaFranchi, S: Adolescent thyroid disorders.Adolescent Medicine 13: 2002

32. Kempers, MJ, Lanting CI et al. Neonatal screeningfor congenital hypothyroidism Based on thyroxine, thyrotropinand TBG: Potentials and pitfalls. JCEM 2006;91:3370-3376.

Page 299: Original Changes to Medstudy 2010 Getting Re com

33. Update of Newborn screening and Therapy for CongenitalHypothyroidism. Pediatrics 117:6. 2006

34. Sisto, P: Endocrine disorders in the neonate. Pediatric Clinicsof North America 2004; 51:4.

35. Levine, L: Congenital Adrenal Hyperplasia.Pediatrics in Review 21:5. 2000

36. Kolon, TF, Hyun, G: A practical approach to intersex in thenewborn period. Urological Clinics of North America 31(3).

Bibliography

newborn period. Urological Clinics of North America 31(3).2004

37. Rosenfield, R: Identifying Children at Risk for PolycysticOvarian Syndrome. Journal of ClinicalEndocrinology and Metabolism.92:3. 2007

38. MedStudy Pediatric Core Curriculum 2010

39. PREP Content Specifications 2010

40. Thomlinson,GE: Genetic predisposition and screening in

pediatric cancer. Pediatric Clinics of North America - Volume49, Issue 6 (December 2002),